You are on page 1of 101

EMREE

Manual

Emirates Medical Residency

Entrance Exam Manual

2020 - 2021

1st edition

By Docterule Team

My advice

Firstly, Congratulations that you have reached to this step in your medical career

and wish you all the success in your future.

Resources

No unified resource for studying, you can choose whatever suits you e.g.:

First aid CK

Master the Board

Kaplan Lectures Notes

USMLE secretes step 2

Toronto Notes

Many doctors asks about other Qbanks such as UW or Canada Qbank or pretest

series, none of them is a fixed source for exam questions, EMREE exam structure

differs completely from USMLE , PLAB, MCCQE2. If you did any of these exams

or practice their Qbank for sure it will help you strengthen your information, but

none is required for the exam and you can not rely on them or the exam. Solving

the EMREE Qbank is a MUST to preform well in the exam.

Exam Structure & Difficulty

Exam is not difficult at all, you need only to study the Q Bank very well and go

over the high yield topics

Exam timing is 3 hours. Usually it is advisable to take the 1st attempt in December

(1st EMREE in each year) as usually questions are easy and repeated from previous

years (but this is not a rule)

Repeated questions in the exam has different percentage from time to time,

sometimes 60% others 90% ,based on that try to solve questions by yourself and

get the evidence behind each answer.

Exam preparation

Preparation time for the exam differs from doctor to another based upon the

knowledge base. It may range from 2 weeks to 1 month.

Try to be clever in studying:

Qbank take a look at it and try to find the correct answer for each if you was

not sure about it.

Strengthen your weak points by studying them simply

Exam Results : Percentage or percentile!

Your percentage is important, try to get as high as possible, repeat the exam

if you wish ( 2 attempts allowed if you pass from the first time). Scoring

high in the exam will give better chance to get multiple interviews in

different places.

Percentile is for you, nobody will look at it.

My personal study plan ( 2 weeks duration):

I tried to solve all the Q bank and verify the answers, I googled the

confusing questions, ( I mean using Medbullets, uptodate or amboss) not

essentially usual google, if you find the answer in any of the references well

and good but many questions you will need to use google (scholar).

them and ask seniors (consultants, specialists), or you can ask in telegram

group (may be someone has evidence behind it).

After finishing all the Qbank for the 1st time, repeat it again from the

beginning as a revision. Then if you have time flip the pages of your

reference to have a look for tables or pictures.

Personally, I used First Aid CK for my first attempt, and USMLE secrets for

the second attempt , My weak points were in pediatrics and OBG so I tried

to watch Kaplan videos along with first aid in order to understand the topics

your time watching or reading everything, just your weak

point).

For basic sciences questions they are not that many, maximum it will reach 5

questions ranging between microbiology , biochemistry and anatomy.

Finally! Take good rest before the exam, sleep well and take your breakfast so

you can score well.

In this manual you will find:

All the questions papers from previous years categorized based upon the subject,

you will find any repeated questions although I tried to remove but still.

Answers here are not the correct key for all the questions, many of them are correct

but maybe not all of them

Special thank for all who shared their recalls.

Contains all recalls till exam of 22 February 2020

Wish You All The Best

Docterule Team

Contents

1. Anesthesia

2. Anatomy

3. Biochemistry

4. Biostatistics and epidemiology

5. Breast diseases

6. Cardiology

7. Cardiothoracic

8. Dermatology

9. Electrolytes imbalance

10. Endocrinology

11. ENT

12. Ethics

13. Family medicine

14. Forensic Medicine

15. General Surgery

16. Genetics

17. Gastroenterology

18. Hematology

19. Infectious diseases

20. Immunology

21. Microbiology

22. Nephrology

23. Neurology

24. Obstetrics and Gynecology

25. Occupational Medicine

26. Ophthalmology

27. Orthopedics

28. Pathology

29. Pediatrics
36. Trauma

30. Pharmacology

31. Psychiatry 37. Urology

32. Pulmonology
38. Vascular

33. Radiology

34. Rheumatology

35. Toxicology

Anesthesia

Anastheisa atient with flial chest, intubated and sent to ICU, what's the

best method to evaluate his plmonary function? End tidal co2

Anatomy

Upper Limb

A man falls on his outstretched arm. Now he has weak dorsiflexion, sensory loss

over the dorsum of the thumb. What nerve injury is it?

Radial nerve

Case of humeral fracture affecting extension of wrist and fingers + sensory


unalinedan nervesaided
disunion

loss of sensation of thumb and forefinger on dorsum of hand. Which umar radialn wristdrop

n clawhand
nerve is affected?

- Radial nerve

- Median nerve

- Ulnar nerve

A woman was working on her computer for too long.

She has throbbing pain in her hand and wrist, with loss of sensation on the

medial area of the hand. Dx? middle 3 fingers

Median nerve

Lady has hand deformity after trial to cut herself ( suicidal attempts ) , picture

was showing ulnar claw hand?

Loss of little and ring finger extension at the MCP joint

Head and Neck

aphonia after thyroid surgery which nerve? Recurrent laryngeal

nerve

What makes esophageal cancer untreatable: recurrent laryngeal

nerve involvement ,Involvement of para carnial lymph nodes

Irritation to the phrenic nerve will cause pain in the shoulder , what is the

root of the phrenic nerve C 3- 5

Man with drooping of right eye and cant frown and no sweat Which nerve

involved

o Right facial n

o Left facial n rt sympathetic

o Right sympathetic chain

o Left sympathetic chain D

o Right Oculomotor n** Horners Syndrome

Picture with right eye ptosis, which nerve is affected :

a- Oculomotor

b- Facial

c- Trigeminal

Mass on side of face cannot blow whistle which nerve Facial nerve

Picture of A man presented with a 2 day history of flu like symptoms after which

he developed some facial weakness. When asked to smile, this is what showed (a

picture of a man smiling with only the left half of his face). Which nerve is

affected? Right Facial nerve

Lower limb

Athletes has an injury to his ankle above the level of medial malleolus , he

developed loss of sensations at the sole and loss of flexion of the toes and loss of

inversion , which nerve will be affected ?

Tibial nerve

Peroneal nerve

Medial planter

Lateral planter

Sural nerve

Patient with high stepping gate. After a car accident injury of the

cerebellum fracture of hip and both femurs.

What injury can lead to this problem?

Common peroneal Nerve,

Tibial nerve,

popliteal nerve,

Cerebellum lesion

Abdomen and Pelvis

Anatomy Lady needed pudendal nerve block by pethidine in her

instrumental delivery . What are roots of puodendal nerve ?

- S1, S2, S3

- L1, L2, L3

- L2, L3, L4

- S2, S3, S4

Guy with some injury at T10 , what is your finding?

Loss of abdominal sensation

T10-T11 compression . What will happen ?

- Loss of sensation over abdomen

- Lower limb weakness

Biochemistry

Patient with peroxidase deficiency. What substance will be accumulated

acid chain

o VLDL

o Ketone body
fatty
Very long

o Long chain fatty acid

o Long chain fatty acid

The rate limiting enzyme for dopa synthesis :

o Hydrozylase Tyrosine
hydroxylase

o Decardoxylase

Biochemistry Which amino acid is in urea cycle

o Leucine citrulline
line

o Glutamate Acid ortho

o Proline Amino cycle Arginine

o Ortholine in urea

Biostatistics and epidemiology

Study Designs

A study done measure the prevalence of obesity among teens in alain.

They choose randomly 1800 student from 8 different school and study

their age,gender and family income. They measure also their body weight

and height. What is the type of study they use?

o Cohort,

o case control,

o cross sectional,

o experimental

Group want to do study on skin cancer and sun exposure, they took

random patients of Bedouins, and distribute questionnaires about sun

exposure hours and skin health, what the type of this study ?

o Cross-sectional study

2 groups smoking and non smoking followed for 10 years which study ?

o Cohort

A study enrolled two sets of people- smokers and non-smokers and

assessed their risk of developing cancer in x number of years. What is this

type of study?

- Cohort

- Case-control

- Randomized control trial

- Cross-sectional

Study studied the incidence of lung cancer in2007 then at 2017 ?Cohort

To compare two studies done 1998 and 2003 about melanoma delayed

diagnosis. What is the best method to compare the result?

o Chi-square test,

o Independent T test,

o Histogram,

o Correlation coefficient ,

o Paired Ttest

Experimenting a new diet in two groups of randomized people .which

type of study

o Experimental study,

o cross sectional study

Someone wants to conduct a research, gave experimental stuff to first

group and nothing to the second group. Type of the study

o Experimental,

o cross sectional,

o observational,

o cohort

Study took a survey regarding previous history of smoking in patient with lung

cancer Case control

Some study about Smoking in teen +18 and a definition of demand was given

and then question about that study?

Risk factor

the highest form of evidence is meta-analysis.

Bias and study error

Medical university, want to do a trial comparing new innovation

procedure to others. After consent form, patient received trials randomly

and the outcome were outlined ?

o Selection bias and confounding factors are reduced,

o Decreased sample size

A study conducted to assess healthy work environment. The sample was

immigrants working in construction. What type of bias?

o Recall bias

o b. Selection bias

Evaluation of diagnostic test

Screening for juvenile stomatitis : what makes it good/valid screening?

o Same result every time ,

o Detect several diseases in one process ,

o Uses multiple processes ,

Quantifying risk

a study was conducted to assess correlation between chewing tobacco

and incidences of MI.

An odds ratio of 1.14 with confidence value of 0.8-2.3

What is the interpretation ?

o Null hypothesis cannot be rectified

o Chewing tobacco causes MI

o P-value required

o D) Recommended t-test to interpret

mber of

patient was 200 , then the incidence in the first year was 20 , then after 10 years

the incidence came to be 90 or something like that

In short learn how to calculate the cumulative risk , answer was 55%

Statistical Analysis

In order to make the results least to be due to chance the p-value should be:

a. 0.99

b. 0.95

c. 0.50

d. 0.05

e. 0.01

Intercept A small table was providing numbers ranging from -1 , 0 , +1 for

different variables which includes level of education , socioeconomic status and

their relation with Gestational age

Then in the bottom of the table they write intercept = 1.000 sth

R2 = some number

o Mean

o Median

o Mode

o Intercept at zero

I could recall any of the option but it was around these things

Study was conducted to assess correlation between chewing tobacco and

incidences of MI. An odds ratio of 1.14 with confidence value of 0.8-2.3 What is

the interpretation ? Null hypothesis cannot be rectified

Study was done in FMHS among European and Asian doctors regarding

cardiovascular disease. European doctors were found more at risk of the

cardiovascular disease that Asian doctors. The next step in this study is

o analysis the data for confounding variables,

o start program for the European doctors to decrease the risk of CVS disease


Breast disorders

Diagnostic Algorithm

54 years old having left upper quadrant breast mass (non tender), it was

not fixed to the skin. Mammogram and US showed micro-calcification

and suspicious mass. How to confirm the diagnosis FNA, Excision

biopsy, Incision biopsy, Modified mastectomy

52 years old female, requested investigations to make sure that she does

not have breast cancer, most sensitive and specific test FNA

50 year female breast mammogram show micro calcification no palpable

mass, suspicious cancer, next management FNA

50 year female breast mammogram show microcalcification no palpable

mass ,suspicious cancer, next management mamo 3 month or excisional

biopsy, FNA biopsy

patient that did a mammogram and they found out microcalcification.

answer is to do a FNA biopsy

A 42 year old woman reports to the surgeon that she is worried about

a lump that she feels the right breast. The surgeon observes a 2 cm by

3 cm mass in the right lower quadrant of the breast. There are no

associated skin changes and the mass has limited mobility. There is no

discharge from the nipple. There is no axillary lymph node

enlargement. Examination of the left breast and axilla was completely

normal. A mammogram report suggests the presence of

microcalcifications.

Which of the following is the most appropriate next step in the

management of this patient?

o Observation for one year and repeat the mammography

o A needle-guided biopsy of the breast

o Excision biopsy of the breast

o - Partial mastectomy

A lady presents with microcalcifications on her

mammogram. Next step? Excisional biopsy

Patient has micro calcifications suspicious of cancer but no palpable mass

on exam. Choices were :

o repeat mamo in 3 months ,

o repeat in 12 months ,

o do excisional biopsy ,

o give tamoxifen for 6 months

Breast abscess

A 26 year old female she has delivered a normal baby. She is lactating.

She developed a painful large mass in the right breast. It was warm. What

you will do as a first step management

analgesia and antibiotic,

Incision and drainage

Breast a lactating lady with 4×4 cm fluid filled cyst in her inner right breast

o I And D

o Antibiotics

Mastitis

47.female with breast engorgement, tender..

A)breast abscess

B)mastitis

lady having fever, tender breast, overlying skin inflamed. treatment

antibiotics , incision and drainage

Lady, breastfeeding, symptoms of mastitis. What to do? Broad spectrum Abx

and analgesics

Patient postpartum presented with fever one side breast pain ,

examination reveals tenderness and erythema of skin. Looks like

acute. What the first step ? (repeated)

- Broad spectrum antibiotics and observe

- Incision and drain

Engorgement

Woman in postpartum period present with fever, breast fullness, what is

the diagnosis Breast engorgement, Mastitis, Breast abscess

A feeding mother presents with postpartum bilateral breast fullness and

pain. Fever 39, Diagnosis? (no redness or erythema mentioned)

A) Engorgement

B) Mastitis

C) Breast abscess

Female presented 3weeks post delivery with bilateral breast pain and no rash

what is the cause fever of 39, feels unwell and tired

a. Breast engorgement (39 fever is acceptable)

b. Mastitis

c. abscess

Female presented post delivery with bilateral breast pain and no rash what is the

cause

a. Breast engorgement

b. Mastitis

c. abscess

Breast discharge

Female Patient with infertility for 2 years with breast discharge ,

prolactin 90, TSH 7 what to do next ?

A) T3 & T4

B) CT of sulla turcica

C) bomocreptin

D) ???

Breast Cancer

A 55 year old female patient present with blood discharge from the

nipple. What is the most likely diagnosis ductal carcinoma, ductal

ectasia, paget disease, pue de orange

The commonest presentation for breast malignancy is in:

a. inner upper quadrent

b. inner lower quadrent

c. subaerolar

d. outer upper quadrent

e. outer lower quadrent

What increases the risk of breast cancer in a woman?

- History of great aunt passed away due to breast cancer at 60

- History of mother passed away due to breast cancer at 60

- History of mother passed away due to breast cancer at 45

Cardiology

ECG

ecg picture of ECG with monomorphic PVCs Options Afib , Extracystole

Scenario and ECG picture displaying rhythm from lead I,II,III what is the

interpretation Extrasystole, LVH, Atrial fibrillation, AV block

ECG showing third degree AV block

ECG showing ventricular tachycardia.

Ecg with AF

Ecg wit

Arrhythmias

Patient with ECG showing hidden p wave ..what to give after cardioversion

o Adenosine

o b. Amiodarone

Patient in ER, LOC, ECG Vtach IV amidarone , Cardioversion

Patient , pulseless , ECG shows ventricular tachycardia

o Epinephrine

o Amiodarone

o lidocaine

o magnesium

A child who is not responsive with GCS of 6 or 8. Pulse 220. With BP

60/40.ECG showed narrow complex tachycardia. What is your next

step Cardioversion, vagal stimulation

Child not responsive, hypotensive, cold extremities in narrow Vtach .

Management

o A.Synchronised cardioversion

o B.vagal maneuvre

o C.digoxin

PSVT tx-

o adenosine

30 years old female who present to the clinic with dizziness and

headache. ECG was shown ( ? Atrial fibrillation ) what is the best

management Refer to ER, Refer for out-patient cardiac clinic

78 years old came to the PHC because of palpitation, dizziness with

abnormal ECG Send him back home, Order 24 hours ECG, Referee to

emergency department

10

A 68 years old patient came complaining of palpitation for 7 days. ECG

is showing atrial fibrillation. Next appropriate step is:

o Cardiovesion

o Anticoagulate

o Rate control

(You have to anticoagulate any patient with AF if more than 48 hours )

patient with a known history of hypertension for the last 10

years, presented with history of loss of consciousness. ECG showed: left

bundle branch block with prolonged P-R interval: 0.34 sec. The most

likely underlying pathology is :

o Multiple and transient pulmonary embolism

o Orthostatic hypotension

o Complete heart block (strokes-Adams syndrome)

o d. Paroxysmal hypertensive encephalopathy

A case of absent P wave.Patient had palpiation. After failed rounds of corecting

it, what to do next

o Amiodarone

o b. Adenosine

child with bradycardia he was given a : BB

Heart failure

Patient had MI, on Aspirin and Lisinopril, also had arthritis on

naproxen for 2 weeks, complication of adding this drug

o water retention,

o Renal stones,

o hemorrhagic stroke

Old lady known case of DM2 , HTN on medications , she is

having joints pain so took her son's diclofenac TDS for 2 weeks ,

her pain improved but developed SOB , leg edema and crackles

on lung auscultation . What to do ? (From 28 Jan)

o Stop diclofenac

o Start fesinopril

o Change her oral hypoglycemic

Female that is taking nsaid for joint paint and she is on B blocker for her

hypertension and her BP is not controlled. what will u do? change nsaid to

paracetamol (because Nsaid causes water retention

Patient is diabetic, HTN, and using NSAIDS or requires NSAIDS for

11

give paracetamol

Lady after surgery given fluids, 6hrs later developed B/L lower limb edema and

sacral edema. Likely cause? Fluid overload

60 year old male, hx of HTN and DM, brought to ER c/o SOB when

going upstairs, he is on Aspirin, ACE-I sulfonylurea, B-blockers, his

pulse was normal, BP normal, RR normal, ECG normal, cardiac enzymes

normal, what further drug you want to add

o digoxin,

o losartan,

o morphine,

o heparin,

o streptokinase

Which of the following is not a risk factor for heart disease : systemic

hypertension ,Smoking ,Uric acid rise ,Decrease HDL

Case of Heart failure, basal lung crackles, no lower limb edema

o Right sided heart failure

o Left sided heart failure

o Compensated heart failure

o d. Combined heart failure

Jvp waves were given along with a scenario of cirrhosis and varices as well,

question was which of the feature of the wave was the cause for cirrhosis,

keeping chronic high right atrial pressure in mind i chose , Extra large a wave

Pulmonary stenosis

Patient with heart failure , hardly compensated , what diuretics not to give ?

o Mannitol

o Thiazide

o Amilroine

o Sphrinolactone

o Erthyoic acid

Cardiomyopathy

Young male, SOB, CXR: left ventricular hypertrophy, +ve FHx of

sudden death Hypertrophic cardiomyopathy

A young footballer presents with with episodes of SOB. Cough and light

12

headedness. Episodes occur solely during exercise. Normal ECG.

Next investigation?

o PFT

o CXR

o ECHO

Coronary artery disease

22 years old , his doctor is suspecting coronary arch disease , what is the

diagnostic step ?

o CT angio

o MRA

o Echo

asymptomatic

o Stress ECG

o Echo

o Cath

Man undergoing endoscopy .developed chest pain .ECG show MI

.management?*take him for angioplasty (if it says best management )

*nitrates ( if it says next step)

what you will do?

o Troponin test and refer to cardiology according to result***

o Admit for further test

o Echo

o d. Anticoagulated

man with recurrent chest pain that lasts around 10 mins plus he had hypertension

and suspected pulmonary edema. So which drug would you choose to treat his

chest pain, hypertension and pulmonary edema.

o nitrates**

o ACEI

o nifedipine

o Furosemide

o clopidogrel

13

The current 2014 ACC/AHA NSTEMI and 2013 ACC/AHA STEMI Guidelines

as well as the 1999 ACC/AHA Expert Consensus Document all support the

following to be contraindications to the use of any form of nitroglycerin include:

Hypotension (usually reported to be a systolic blood pressure < 90 mm Hg) or

a > 30 mm Hg drop from the patient's baseline

Bradycardia (< 50 beats per min)

Tachycardia

Patients experiencing a right ventricular infarction

Use of avanfil, sildenafil or vardenafil within 24 hours, or tadalafil within 48

hours, due to the risk of significant hypotension and/or cardiogenic shock

Note: Morphine can dilate the venous system and result in a reduction in

preload, which can worsen tachycardia (thereby increasing oxygen demand) and

patients with right sided MIs since they are preload dependent. Giving nitrates to

patients who have recently taken type 5 phosphodiesterase inhibitors (e.g.,

sildenafil), prevents the breakdown of cGMP and cause more profound

vasodilation.

Old patient with chest symptoms for month then resolved. Troponin negative.

ECG given.

o STEMI

o NSTEMI

o Unstable angina

o Non cardiac cause

Patient post MI , the most common possible complication ?Arrhythmia

patient with chest pain for 16 hr., ecg normal. Best test for initial

management?

- ECG ( if not repeated since 16 hours, repeat it )

- exercise ECG

- Troponin

Congenital heart diseases

VSD

A)increase in pulmonary vasculature

VSD , increased peripheral resistance

VSD child advices

o he will hv recurrent SOB , harsh sound is positive sign

14

pulmonary hypertension )

Young child with ejection systolic murmur 2/6 , increased

pulmonary vasculature and dilated right atrium and ventricle .

What is the most likely cause ?

- ASD

Pediatric patient with a systolic murmur investigation showed right ventricular

and atrial hypertrophy ?

o ASD

o VSD

o pulmonary valve stenosis

Dyslipidemia

Patient with a father that had CAD, his LDL is 400, triglycerides level is

combined

hyperlipidemia

young man presents. his father died when he was 50 . LDL HIGH HDL

low TAG high which type ?

o familial

o combined

o type IV

28 years old gentleman presented with xanthomas in hands and body , his father

died at 51 due to an MI , Lipid profile shows : Cholesterol 400 (high) , TAG

(high) : 200 , HDL (low) : 45 . What is the diagnosis ?

- Familial combined hyperlipidemia

- Familial hypercholesterolemia

- Type IV dyslipidemia

Hypertension

Nifidepine

(CCB)

Hypertensive patient on BB uncontrolled which medication to add for

Nifidepine, Duritics, Nitric glyceride

HTN, chest pain and pulmonary edema. Which drug will treat all of them?

a) nitrate

A 40 years old lady presented for routine check up in the PHC. Her blood

pressure reading today is 150/90 mmHg. No previous records are

any medication. The best next step in management is:

o start her on low dose B blockers

o re-examine her blood pressure in 3 months time

15

o do ECG

o d. check serum electrolytes

old patient with one reading of BP : 144\92 , he is not visiting any doctor

since long time, what u will do for him

o Reassure him that this is normal for his age ,

o Repeat measuring bp after 6 hours ,

o Give him bblocker

o ,Give him CCB

football player , His bp was 140/85 , What to do next o Should we call

him next week to measure his BP

o Should we not call at all

o Should we conduct other test

High BP in a young man trying to join school team.

o come after 1 week

o do echo

Depend in a scenario if there is syncope the Echo

If no syncope A

Lady, came to ER with swollen leg, diabetic and hypertensive, her bp is

160/100 , on glyburide , tranexamic acid for DUB,

Which of the following is most linked with her current condition?

o uncontrolled htn

o Prolong use of tranexamic acid

o Glyburide

Malignant hypertension

patient with hypertension came with pulsating showed infrarenal

aortic aneurysm abdominal mass.he is on lisinopril. examination showed

high bp and ultrasound showed infra renal aortic aneurysm 4.5 cm in

size with mural clot.what to do nxt

o a)surgery

o b)anticoagulant

o c)increase lisinopril

o d)stent

o e)follow up

Valvular heart disease

16

your diagnosis mitral stenosis, aortic stenosis, pulmonary

hypertension

Patient with valve problem, underwent surgery and had artificial

For life if

mechanical, 6month, 3 months if biological

patient with a known systolic murmur, with dyspnea and

shortness of breath and chest pain who recently developed episodes of

loss of consciousness. Neurological exam was normal. The most likely

underlying pathology is:

o Mitral Stenosis

o Mitral regurgitation

o Aortic stenosis

o Aortic regurgitation

Man with syncope, SOB since 2 days, mild angina on and off since 2 months

with Harsh ejection systolic murmur. Treatment is:

o Admit for further test

o Do troponin and then refer accordingly

o Order OPD Echo.

o ans: c

Pt with harsh murmur and said about mild angina 2 months ago what to do :

o Order tropinin

o Admit him for more investigation

o c. Order echo after one week as an outpatient c

Patient after a fainting spell he came to the ER, There was a murmur that radiates

to the neck (signs and symptoms of aortic stenosis), coming in with heart failure

they asked for further management ?

o admit for further test

o do echo opd

o trop

o d. Troponin and refer to cardiology accordingly

Pt with HTN, rheumatic disease with holosystolic harsh murmur in LLSB

17

o Inverted P wave in V1

o Large p wave in lead 2

o Wide QRS

o d. Prolonged PR

or b

Cardiac Pharmacology

50 years, lower limb edema, on Lisinopril, amlodipine and statin,

which causes her symptoms Amlodipine

Drugs that does not cause dyslipidemia ACE inhibitors, Beta blockers,

diuretics

highly selective for postsynaptic

alpha1- adrenergic, and non-selective for beta-adrenergic receptors

One guy with HTN was on hydrochlorothiazide. His BP high. Doctor not

happy with is so he changed it to furosemide. What electrolyte will you

have monitored closely K, NA, CL, CA

One patient having K of 5.5, which diuretic in contraindicated in his

situation Spironolactone

Other topics

Coartication of aorta

What is the most important cardiac abnormality is present in pregnant

Coartication of aorta,

VSD, Most common is ASD

80% stenosis with symptoms of TIA what to do next ? endarterectomy

A patient came with chest pain and a CT scan showing aortic dissection

Case of woman that comes with her boyfriend who was a relied history giver.

She complains of lightheadedness, fatigue, this has never happened to her before.

She thought it was because of work. What to do for her?

a. Hydrocortisone + IV hydration

b. Dextrose + IV hydration

c. Toxicology screening

Cholesterol emboli syndrome

18

In medical resuscitation for something , what is the best way to measure the

respond to resuscitation ?

o Urine out put more than 40 **

o Central pressure measures

o Heart rate more than 100

o Systolic blood pressure

infective endocarditis cause CVA

Patient has a tender chest mostly at the costochondral junction Costochondritis

Patient has bleeding and receive blood transfusion then had crackles

o Fluid overload

Patient with coronary vasospasm (atypical angina vasospastic), how prevent that

o Diltiazem

o Propranolol

o Nitrates

Young male presented pain in the left sternal border . He is exercising 3

times/week with friends . He smoke cigarette as well as shisha . He is under

a lot of stress because of his study . He reports that pain increases with deep

inspiration , pulling or pushing against wall with his arms . On examination

lungs are normal . What is the diagnosis ?

o Chostochondritis

o T3-T4 disc herniation

o Esophageal spasm

thoracic USG chest was provided , with IVC dilated and low BP , what is the

cause Pseudo low BP

-mitochondrial DNA positive

treatment ursodeoxycholic acid

Patient with history of Rheumatic fever , come to you now with

A grade 2/6 systolic ejection murmur increases when you bend forward. What

Ecg change would be correlated with patient presentation right now ?

o diffuse ST elevation.**

o ST depression.

o PR interval elongation

This case is started with RF , causing pancreatitis , once the carditis (myocarditis,

pancreatitis) happens on RF , PR interval would NOT count as minor criteria.

And the patient present with pancreatitis and the ecg changes with pancreatitis is

Diffuse ST elevation. a

19

Cardiothoracic

Trauma cases

Patient brought due to road traffic accident. On examination,

muffled heart sounds, distended JVP. Chest was clear on examination

o Pneumothorax

o Cardiac tamponade

o - Hemothorax

RTA with flail chest dull distended abdomen, patient intubated and saturation

improved, next management

Patient with trauma. What is the most life threatening

condition Tension pneumothorax, epidural hematoma

Patient had RTA with multiple fracture, trauma to abdomen and

pneumothorax

Case of patient with trauma to chest, abdomen and pelvis. Decrease

breath sounds on one side with dullness on percussion, what Is your

Mx Insert chest tube

Patient presented with GCS of 8. On examination there was unilateral

hemithorax with reduced air entry. And hyperresonance on percussion what

intervention

o ICU and observe

o Tube insertion

o Oxygen

o b. Tube insertion

Cardiac diseases

Patient with hypotension, muffled heart sound and distended neck veins

Cardiac tamponade

Thoracic diseases

Inserting central venous catheter, patient developed hypotension,

tachycardia, distended neck veins Tension pneumothorax

Post cardiac transplantation, had some pneumonia, and culture showed normal

flora. What could be the cause if he did not improve with antibiotics for 4

months?

o Aspergillus

o Mycoplasma

o Nocardia brasiliensis

o CMV

o e. mycobacterium avium

20

chylous material was aspirated from pleural effusion, what is the cause?

o TB

o Lymphoma

o Trauma

o d. CHF

Man with sudden onset of chest pain while exercising :

Spontaneous pneumothorax

Smoker young male , had pain just left to sternum, able to maintain

weight lifting , pain increase with deep breathing and pulling or pushing

Dx Chostochondritis ,T3-T5 prolaps

Mediastinal diseases

Presentation of frontal neck mass , moves with deglutition Chest X Ray

shown , Dx

o widened superior mediastinum ,

o Mid mediastinum widening ,

o Unfolding of aorta

chest x-ray showing widened mediastinum. I think it was enlarged

enlarged

mediastinum

21

Dermatology

Dermatitis

A male photographer developed white patches on his both forearm more

than 8, small in size, his rash appears post working in his photographic

film small room in his home, his sister has asthma, his brother has

allergic rhinitis, when he eats shellfish the rash increase, what is the

diagnosis

o Allergic contact dermatitis,

o Endogenous eczema,

o Scabies,

o Leukoderma,

o Contact dermatitis

Male patient came with itchy skin rash at the area of his belt, Picture

Another case with similar scenario but asking about the management ?

o Steroids cream

o Antihistamine

o Emollient cream

A 16 years old boy had an onset of papules and vesicular eruptions on the

face and arms after having a walk in the park. He had similar complaints

last year when he walked in the same park. The most likely diagnosis is:

o Atopic dermatitis

o Acute contact dermatitis

Psoarsis

o Topical steroid**

o Ketoconazole

o Cephalexin

o Aluminum hydroxide

Psoriasis Q :Patient with lesions in both of her legs about 10% of body surface

,Asking about tx

o Topical mometasone + oral prednisolone

o topical 2 things

o Topical emollients

22

o UV

Psoriasis: 10% skin area affected. Axilla affected. Koebner phenomena seen.

How to treat:

o Topical clobestasol + calcipotriol

o Topical mometasone+ something

o Oral steroid

o Efalizumab S.C injection weekly

o Topical clobestasol + calcipotrio

Acne

Patient with acne all over the face with papules and pustules what is the

Clindamycin

young man with skin acne. he tried many over the counter medications

and all failed. which drug to give. Oral retinoic acid

( Answer will depend on the type of acne lesions

Skin cancer

A 68 year old Bedouin man, complained of a small lesion in his nasolabial fold

that was paler than his skin, it was not bleeding. However he tried to poke it and

ever since it formed a non healing ulcer. What is the diagnosis ?

o Malignant melanoma

o Basal cell carcinoma

o c. Squamous cell carcinoma B

BCC features in man with blue eyes and fair skin.

Cancer meta skin

Farmer with Bleeding mole 4cm macule and ulcer in middle, next step?

o Shave biopsy

o Punch biopsy

o Ans: BCC, a

Lesion on the upper back of the patient, color grey blue, flat, irregular border,

bleed sometimes

o Malignant melanoma

o BCC

o Solar keratosis

23

o D) ?naevi

o Malignant melanoma

indicating poor prognosis

o Size more than 6 mm

o Thickness

o Color variation

o Irregular border

Smoker man with painless lower lip ulcer . Dx? Squamous cell

carcinoma , Herpes

Others

A lesion on shin, Began as nodule. Turned into deep ulcer with necrotic

tissue and violaceous borders

o Pyoderma gangrenosum

o Erythema nodosum

o Pretibial myxedema

o Necrobiosis lipoidica diabeticorum

A man with Lesions in axilla sacral and groin areas, he was being treated for

tinea for 4 months with no improvement. Erythematous base with scales, except

in the groin area where it was moist. KOH showed no hyphae.

o Lichen planus

o Psoriasis

o Candidiasis

o Secondary syphilis

Child presented with rash that is pearly in shape , school nurse sent him back

home because he is contagious Molluscum contagiousm

CKD +DM type 1+ scratch dry rash on legs and on back? Which medication?

tiny scratches

o Gapen..

o Oral tacrolimus

o Topical tac

o topical Emollient

Itchy Lady complained of a rash on her face. No acne history when she was a

teenager, What is the diagnosis? Picture provided.

o Acne vulgaris

o Acne fulminans

24

o Folliculitis

Lady with confined lesion to one area on her forearm:

o herpes zoster

o Eczema ,

o Scabies

man working in a cupboard/wardrobe furniture making shop...has slight cough

and nose irritation which resolves on the weekend...now patient has cracks on

hands and feet

o Hard sawdust

o Organic paint stripper

Patient with small scaly patches on chest and back..hypopigmented

o Tinea versicolor

o Tinea corporis

Tinea Corporis:

a. Scaly pruritic eruption with sharp irregular borders with central

clearing

b. Investigation ; koh showings hyphe , fungal culture

c. Ttt ; topical oral antifungal Clotrimazole

Ptyrisis versicolor

a. management of P. Versicolor :: topical ketoconazole/ Selenium

sulphate

Woman with well circumscribed annular rash with central clearing and scales on

the periphery for the first time. These lesion are present over the arm trunk and

pustules ; vesicles and plaques

How did she get it?

o contact with animals

o Contact with chemicals

o New drug reaction

o Allergic

o ecent history of bacterial infections contact Cwith animal

three lesions on hand a lot of keratosis happening

o Merkel

o Seborrheic

o BBC

o Sq Ca

o 5. Keratoacanthoma

25

Spider angiomata picture

Rash between web and on extensor surfaces and there is a line or track under the

skin (scabies) = Permethrin

26

Electrolytes disturbances

Sodium disorders

patient with hypernatremia, why does he only have mild symptoms of

dehydration. Answer is because the sodium retains the fluid in the plasma

(extracellular fluid accumulation

because the sodium retains the fluid in the plasma and

thus he does not seem to be depleted

Patient with hypernatremia:

a. Increased oxybutynin.

Potassium disorders

Female had persistent vomiting, what electrolyte change she will have?

Hypokalemia

female keeps vomiting what electrolyte abnormality will she have.

Hypochloremic Hypokalemic metabolic alkalosis

Same scenario of the tourist that was lost in the desert and walked alot .She was

the dehydrated patient with the ecg

a. Hypokalemia

b. Hyperkalemia

c. Hyponatremia

d. Hypernatremia Hyperkalemia

Peaked T wave

Flat p wave

Acid Base Balance

ABG values given

A.Respiratory acidosis and hypoxia

Calculate A-a gradient 30

The Alveolar arterial gradient (A-aO2,[1]or A a gradient), is a

measure of the difference between the alveolar concentration (A)

of oxygen and the arterial (a) concentration of oxygen.

A-a gradient = PA02-Pa02

PaCO2/0.8 PaO2

27

- PaCO2/0.8

(48/0.8) 60 = 30

A normal A a gradient for a young adult non-smoker breathing air, is

between 5 10 mmHg

Phosphate disorders

Hypophosphatemia with hypercalcemia, frail bone Alp high is the Dx- what is

the lab finding?????

a. Urine phosphate, serum creatinine

Patient diagnosed with hypophosphatasia, how to confirm it?

a. Urine phasphatamines **

b. Parathyroid function test

Infant with hypophosphatemia, hypercalcemia and abnormal digits how to

confirm it?

a. Urine phasphatamines something**

b. Elevated PTH

c. Increase Vit D

Calcium disorders

High calcium symptom

a. Constipation

b. Cataract

c. Trousseau sign a

Low citrate:

Potassium citrate

post thyroidectomy, tingling sensation and musle twitching

IV calcium gluconate

Patient was lost in the desert. His temp 42. And creatinine is very high.

Wat is the cause of high creatinine ( Rhabdomyolysis

28


Endocrinology


Thyroid diseases

52 years old female, asymptomatic, has solid solitary nodule of her neck,

lab results most consistent with this case Normal TSH, high T4, low

thyroglobulin, high antithyroid IG, low T3

follicular neoplasm, next step Lobectomy

26 years old Lady presented with neck mass , FNA done showed follicular cells ,

the lobectomy was done and it showed follicular carcinoma what to do next ?

Thyroidectomy + RAI /Thryroidectomy + external beam radiation

Patient with Thyroid nodule (neck mass) that is affecting her breathing.

Next step in management?

* FNA bilateral

* Excisional bilateral

* Partial thyroidectomy

A 35 years old woman presents with a swelling in the neck. The

swelling has increased in size gradually over the last two years and the

patient feels she has difficulty with breathing. Examination shows that the

mass measures 8cm by 10 cm, soft and not warm to touch. It moves with

deglutition. Which of the following is the most appropriate management

of this mass?

- Partial thyroidectomy

- Oral thyroxine

- Oral propylthiouracil

- Excision biopsy 2 A 46 years

Management of 2x2 cm mass in 1 lobe of the thyroid, papillary carcinoma

( other options : partial thyroidectomy, lobectomy

Thyroid cancer (papillary) what is the management ? Near total

thyroidectomy with radiation

papillary thyroid tumor 2.5x2 cm management?

a. Nearly total thyroidectomy plus radioactive iodine

b. Lobectomy and ipsischemity

Patient has a thyroid nodule showed a papillary thyroid cancer by FNA what to

do next Lobectomy

29

35 years old, 2*2.5 cm papillary thyroid carcinoma whats the management?

Lobectomy and isthmectomy

Thyroid nodules, moves in deglutition, breathing and swallowing

difficulty management thyroidectomy

thyroid nodules .moves in deglutition .breathing and swallowing

difficulty . next step management?*FNAC *partial thyroidectomy

(answer for best managament)

Lady with thyroid nodule, asymptomatic, TSH normal, O/E thyroid nodule 2cm?

what to do next? FNA

60 year female, gradually become isolated, no interest in joining family

gathering or doing her usual activities, talks about herself tearfully,

gained weight, P/E showed obese women with bradycardia and crackles

in the base of her lungs MDD, Hypothyroidism

hypothyroid

Old lady with decreased bowel movements n relative constipation. Hair

TSH

30 YO pregnant lady, known case of autoimmune thyroiditis on Lthyroxin,

she experiences worsening of her Sx: cold intolerance, fatigue.

Her TSH is normal her T4 level is lower normal = 0.8 ( normal is 0.8 to

sth)What is the next step? Reassure her, Increase her thyroxin dose

symptoms on thyroxine. Treatment?

*Reassure her *Symptoms will improve after second trimester

*Increase the dose of thyroxine

Patient presented with a mass in the midline of the neck, the mass was

moving with tongue protrusion, dx thyroglossal cyst, thyroid nodule,

bronchial cyst

A child was found to have enlarged thyroid. All children has same thyroid

problem in his area (?Endemic iodine deficiency)

a. Simple diffuse goiter**

b. Devuion thyroiditis

c. Multi Toxic goiter

d. Graves disease

Hyperthyroidism sympt/ diffuse uptake/ no thyrotoxicosis in labs

a) Subacute thyroiditis

b) Graves

c) Toxic multinodular goiter

30

D) thyroid cancer

Endo Bethsida 3 thyroid , no visible nodules , only you can see them by USG ,

what to do after knowing that it has atypical cells Remove the nodule

Lobectomy

Subtotal thyroidectomy

Weight loss, exophthalmos (big eyes) Graves disease

Patient with palpitations, exophthalmos fine tremors and menstrual

cycles reduced from 5 days to 1 day. Dx?

- Graves disease

- Hyperthyroidism

- Hashimoto thyroiditis

- Toxic nodular goiter

case of graves with a pic of exophthalmos. what are the lab findings.

TSH is low and T3/T4 are high

treatment of thyrotoxicosis

A.Propranolol

Effect of propranolol on thyroid disease affects heart rate, Reverse thyroid

question about hyperthyroidism. unsure of the details but the answer is

fine tremors. which are the tremors u have in hyperthyroidism

post thyroidectomy, tingling sensation and muscle twitching

A.iv calcium gluconate

b.vit d

Patient who had thyroidectomy/lobectomy with postop perioral paresthesia and

twitches. What will you do?

b) ABG

c) serum calcium**

d) other options

thyroid removed develop numbness on face and arms.. which test

Calcium level

Hoarseness of voice after thyroid surgery what is the cause ? Injury to the

recurrent laryngeal nerve

Unable to speak after thyroid surgery which nerve RLN

Adrenal diseases

Patient with symptoms suggesting cushing syndrome: brittle hair, mild

hirsutism, straie, obesity urine free cortisol test, ACTH stimulation

test, ACTH level, loss of diurnal cortisol level

Woman with abdominal obesity, striae, etc? Dx Cushing Syndrome

A lady with symptoms and signs suggestive of PCO (hirsutism, clitoral

31

a. DHEA-serum level

b. Free cortisol level

c. Testosterone levels

d. Dexamethasone suppression test

e. Serum FSH

which lab to measure in congenital adrenal hyperplasia?

A.Free testosterone B.17 hydroxy progesterone

Man , presented with recurrent headache episodes and BP 180/90 sth , his

electrolyte showed NA 143 , K 2.8 , HCO 34 , cl 105 what is the diagnosis

Hyper aldosteronisum

Woman feeling fatigue and other symptoms. Labs show low sodium (124)high

potassium (5.1), glucose 72, BP 80/70 CBC shows eosinophils 12%

what will you give her?

a) dextrose 50% with normal saline

b) IV hydrocortisone + normal saline***

c) other options

Pituitary diseases

Female symptomatic due to low hormones and had previous prolonged delivery

Sheehan syndrome

Patient with headache bilateral frontal and vision problem Pituitary adenoma

Patient presenting with reduced lateral vision bilaterally Pituitary adenoma

13-year-old little girl who fall from bicycle and hit her head. Had CT

scan. She had high Na in serum 148mEq. Complaining of polyuria,

polydipsia, what is DX CDI, NDI, DM

Water deprivation test. Was given medication and symptoms resolve what is that

medication? Desmopressin

A man with confusion and low sodium what is the diagnosis urine osmolality

more than serum osmolality SIADH

Lady presented with small amount of nipple discharge, she is being

amenorrhic for sometime , her TSH is 7 and low T3 T4 , prolactin is 90 .

What to do ?

- Imaging for sella turcica

- Give bromocriptine

- Serum T3, T4 ???

Patient came complaining of disturbed vision, which is affecting her daily

activities. She

manipulation of breast, what is the initial management for her ?

a. Surgical resection of the tumor

32

b. Dopamine antagonist

c. Irradiation of the tumor

d. Dopamine agonist

In the case scenario it was clear that the tumor was affecting her vision to the

extent she had many close calls while driving and was about to have many

accidents, so something has to be done fast other than medical therapy.

tumor size by irradiation to alleviate the symptoms sounds like a good idea. This

One patient has galactorrhea , amenorrhea and nausea and vomiting , due to

some drug , that drug work in which mechanism

o Super sensitivity of dopamine receptors

o Block the receptor for dopamine

diabetic patient Who was taking parkinson medication And metformin, he

develops hyperprolactinemia, what caused it

1. parkinson medication

2. Metformin

3. Dopamine agonist effect

Hyperprolactinemia treatment

a. Dopamine antagonist

b. Dopamine agonists**

Frontal bossing, big mandible, visual disturbance Acromegaly

Frontal pain with vision disturbances. Pituitary adenoma

Parathyroid diseases

Patient presented with abdominal pain , she / he is taking vit D tablets and

exposing to sun excessively , her/his labs : now her ionised and total calcium are

high and low PTH , what is the cause ?

o Hypoparathyroidism

o Vit D toxicity

o Hypothyroidism

o Thyroid adenoma

Diabetes

36 year man diagnosed with diabetes one month back on 5 mg glyburide,

now dose increased to 15mg. He has lost weight, his labs showed random

221, antibodies raised, c-peptide nil, management increase dose of

glyburide, add metformin to glybyride, stop glyburide and start insulin

50 years old truck driver man for 25 years , was diagnosed

with diabetes since years , on Glucophage and controlled .

33

Diagnosed with HTN 2 years back . Now presented with lower

limb weakness and loss of sensation , on examination he has loss

of upper limb sensation . He has jaundice and was scheduled for

cholecystectomy . What is the most likely cause of his sensory

loss ?

- Diabetes

- Prolonged setting

- Jaundice

diabetic lady not copmlaind to meds devolp pnemonia after that came unconcsus

wz abd pain vomting?

o DKA

o Hyperosmolar coma

o Hypoglycemia

Diabetic patient on insulin presents to you for follow-up. Her morning

fasting glucose is 95mg/dL post-prandial 2 hours after breakfast is

180(high) and after lunch is 108mg/dL (normal). What will you do for

her?

- Increase morning dose of insulin

- Increase evening dose of insulin

- Decrease morning dose of insulin

- Decrease evening dose of insulin

Diabetic patient on insulin presents to you for follow-up. Her morning fasting

glucose is 95mg/dL post-prandial 2 hours after breakfast is 180(high) and after

lunch is 108mg/dL (normal). What will you do for her? - Increase morning

dose of insulin

Driver , weakness of the lower limb , loss of sensation in the upper and

lower limb what is the cause DM

What about women with infection, ph 7.35 and glucose 35? Hypoglycemic

coma

Patient came to ER, has history of T1DM, rigid abdomen and tender and

distended, had vomiting also, has low BP and tachycardia, next step

resuscitation with IV fluids, insulin and antibiotics, x-ray of abdomen

Others

Presentation of frontal neck mass , moves with deglutition Chest X

Ray shown , Dx? widened superior mediastinum ,Mid mediastinum

widening , Unfolding of aorta

34

35

ENT

Ear diseases

3 years old female who has 5 attacks of ear pain. This time her

temperature is 39, bulging eardrum & erythematic. What is your next

step in management?

a) Refer to ENT surgeon

b) Access her hearing

c) Grommet insertion

d) Antibiotic

4 years old child with fever , left ear pain rhinorrhea , cervical

hymphadenopathy , mobile ear drum . What is best next step ?

- Augmentin

- Paracetamol

A man with otitis media which nerve affected to cause the facial

presentation, picture of a man with similar presentation in the above

question.

a. Hyperacusis b. decreased lacrimation of right eye c. decreased

lacrimation of left eye d. inability to frown

Nose diseases

Middle age male bleeding for 1 hr (epistaxis), no history other than high BP, 1st

time experiencing bleeding. Presented to the ED with towel soaked with blood,

tried compression and ice but patient was still bleeding. Rhinoscopy was done

out

from both nostrils. A tampon was applied to stop the bleeding which was

unsuccessful. What is the cause?

a) Anterior epistaxis

b) Posterior epistaxis**

c) Mass

d) Bleeding disorder (coagulopathy)

Non allergic rhinitis in female of 5 hrs cough and SOB with runny nose, fever-

what finding on examination---

o Nasal polyp,

o LNs

o ans: a

Throat diseases

Child with recurrent tonsillitis, what is the indication for tonsillectomy

Breathing problem (airway obstruction, absolute ) Recurrent tonsillitis

36

(>3 times per year, relative )

(6 times per year)

>7 per year or >10 per in 2 years

recurrent tonsillitis

Child had multiple sore throat for which he has treated also have

distorted speech .on examination he has large tonsils .indication of

tonsillectomy?

*Large tonsils

* Multiple sore throat

* distorted speech

Child that has several symptoms related to his tonsils and it asks about

ructive sleep

apnea(absolute indication), recurrent infections >3 (relative

indication)

Child with full centor criteria what to give Augmentin

one child with tonsilitis caused by s. Pyogens.

a. Penicillin

b. Amoxicillin + clauvic acid

Neck disorders

Patient with abnormal facial sensation and tinnitus with right sided ataxia

a. Left cerebellopontine

b. Right cerebellopontine**

c. medulla

Right cerebellopontine features

a. Indications include unilateral hearing loss (85%), speech

impediments, disequilibrium, tremors or other loss of motor control.

b. And hypersthesia of the face / diplopia and tinnitus if it was a CPA

tumor

37

Picture of facial palsy which nerve affected

Ethics

A brother of a patient of yours is asking for DNR (do not rescucitate) of

the patient. What do you do then? Respectfully say no to him and continue to

give care

beneficence

A woman is advised to undergo a hysterectomy. She

refused. Dr wants to contact her family members. What is this ethical rule?

beneficence

Female patient who had breast cancer, doctor advised for (biopsy and

surgical/chemo intervention) and she declined. The doctor made another

appointment in order to convince her. What is this the type of

act beneficence, Justice, maleficence, patient autonomy,

Doctor refers a patient to a more specialized doctor ethical principle

beneficence

Non malificience

male patient diagnosed with Hep C. asking the dr not to tell his wife

obably already

have the disease. dr agrees . which ethical principle is this?

Beneficence

Autonomy

Non maleficence

Justice

A 67 year old man who developed a cancer. His son tells you not to

inform his father about the diagnosis as this will make him hopless and

worsen his life.of the ethical principles that aupport the sons request is

beneficiance, non-malficience, confideniality, autonomy

OTHERS I am not sure about answers

Mother with breast cancer , refused biopsy and treatment , the doctor counselled

her son and took consent Autonomy


38

Family Medicine

The Transtheoretical Model (Stages of Change)

45 years old female, DM, HTN, OA in both knees, paracetamol was

given, now she is able to tolerate walking after the tablets, she was

convinced about the importance of the walk for her health and she is walking

for 30 minutes daily, she came to the clinic and expressed that she does not

think that walking will help her much, what stage represent her stage of

change she is now action, maintenance

Patient with cardiac problem , he is chronic smoker , he had to stop

smoking , but he never thought to quit smoking , in which stage he is

Pre contemplation

Well being

History of palpitation and SOB , no chest pain , past history of IBS , what to do

next ?

a) Tropoinin

b) Psychological assessement**

Patient with IBS & tension headache came with palpitation & SOB,

what your next step

a) Assess psychological stress

b) Do cardiac/chest exam

Old patient presented with symptoms of Vitamin D toxicity and lab was showing

Vitamin d toxicity

Old lady post menopause , did DEXA scan for her hip , picture of dexa scan

report was provided , what is the vitamin responsible for this disorder?Vit D

Post menopause lady , presented with bilateral knee pain , worried about having

weak bones , she gave history of 20 years smoking , intermittent inhaled

corticosteroids , last year DEXA +2 , WHAT is your best option to prevent hip

fractures in her condition ?

a) Repeat DEXA

b) Calcium and vit D

c) Alderonate

60 years female, on Ca, Vit D, Elandronate, what does this patient

have Osteoporosis

58 year old male came to the clinic due to trauma to his index finger.

You examine the patient

visit the clinic for the past 10 years. His PMhx not significant. After

39

suturing the wound and giving tetanus vaccine. What is the next step you

want to do as a screen test for this patient ECG, Random fasting blood

glucose level, measuring blood pressure , chest x-ray

16 year old athlete with high bp

repeat measurement after a week

A 53 year old patient with a BP reading of 144\92, he ihas not visited a doctor in

a long time. What u will do about his high BP reading? BP after 6 hrs

old patient with one reading of BP : 144\92 , he is not visiting

any doctor since long time, what u will do for him ? Reassure him that

this is normal for his age ,Repeat measuring bp after 6 hours ,Give him bblocker

,Give him CCB

49 years old African guy requesting for HIV test. He has HTN on medication.

Family history of stroke and HTN. Abdominal aorta could not be auscultated

a) ELISA + fasting glucose

b) ELISA and chest Xray

c) ELISA and Abdominal aorta aneurysm

d) lipid profile and fasting glucose patients with HIV are at risk of having

Tuberculosis so chest X-ray must be done to exclude that

53 year old menopausal woman came to you. She is completely

asymptomatic. Her mother passed away due to breast cancer at age of 53.

What will you give her?

- Calcium

- Estrogen patch

54 yr old lady menopausal b.p 130/90 mm.hg ,no complaints,no family

history, no history of ca.what to give?

A.Calcium supplements

b.give alpha methyl dopa thrice daily

63 years old lady presented for routine checkups , her blood

pressure is around 135/70 , has no symptoms . What will you do ?

- Give calcium 600 mg

A 45 years old man presented in a regular check up in the clinic. He has

hypertension and hyperlipidemia. He is afraid of developing myocardial

infarction. The best advice regarding diet would be:

a. decrease salt intake

b. decrease fat intake

c. decrease protein intake

30 year old female, requesting a good dietary plan, a plan is decided,

40

which component of the following is very well accepted decrease

complex carbs, decrease protein content, increase fat content, increase

simple carbs, increase dietary fiber

a. Health status

b. Health services

c. Risk factor

d. ans: c

e. http://origin.who.int/healthinfo/indicators/2015/chi_2015_65_tobac

co_use.pdf

Prevention health and screening programs

Case of juvenile stomatitis which screening test statement is most

accurate screening result should have same result when repeatedly

done, screening test should be able to find more than one disease at a

time

for the disease to

be treatable and screening can modify mortality and morbidity

been encountered. Although this is a rare disease, you would like to know its

prevalence. To get more information about this, you will approach: a. Local

pediatrics b. Family physician

The following disorder is NOT an identifiable disease to the preventive

medicine:

a. Food poisoning

b. Measles

c. Infectious mononucleosis

d. Chickenpox

proper screening test to be done at this visit Mammography, Dexa,

Random glucose

51 lady who had Aloe Vera allergy , presented to clinic and treated

for that , she didn't present to physician since 15 y . What

screening test to do for her ?

- Mammogram

- Blood sugar

41

- DEXA scan

53 year old menopausal no complaints. Which test to

do Mammography, DEXA scan, blood sugar

55 year lady never went to hospital totally normal before presented this

time because she had contact dermatitis, what is the best thing to check for

her as she never had check-up before: blood sugar, CXR, mammogram

50 years female, controlled DM, coming for F/U every 2 months,

exercise 3 days per week, following diet, did Pap smear last year,

appropriate screening Mammography

Oncology Cancer causing virus.

-8 ( Kaposi sarcoma )

No options were given in the recall but all 4 above viruses cause cancer

Others

18 years old male, came for a check-up because he wants to join the

police force. His RBS = 11. He asked you not to write in the report

that he got diabetes. What you will do?

a) Take hx & discuses his fear & expectation

b) Take hx for DM his factors & do OGTT

c) Refer him to another doctor

Alain municipality was astonished by increasing number of children mortality

because of RTA what measurement you can do ? o Seat belt

o Traffic light

o Air bags for only licensed cars

Study was comparing health burden of CVA between Yemen and

UAE.

a) Disability adjusted and life expectancy

What the most common cause related to high risk mortality in

UAE Respiratory disease Cardiac disease, Cancer , Liver disease

Geriatrics health

96 years old lady came to the family physician for a follow up for

HTN, osteoarthritis. She is a widow living with her daughter who

complains of forgetfulness and losing her way to home. She denies

42

syncope, stroke like symptoms or hallucination. What is the best

diagnostic step:

a) Assess mental status

b) refer to neurologist

c) MRI of pain

d) Refer to psychiatry

A 82 years old male patient was brought to the family medicine because

he fell. On examination he was walking with wide base steps and the upper

limb abducted (as if he is trying to reach something). When asked to turn

around in response to sound, he turn the whole body. What is the most likely

b. Visual impairment

An elderly patient who is in the nursing home for the past 8 years due to

multi infarct dementia. He was otherwise well. Today he was unable to

recognize his caretaker. He refused to take his regular medication. The

most likely underlying pathology is:

a. Acute urinary tract infection

b. Major depressive illness

c. Social isolation

Elderly lady admitted with Myoclonus weakness, incontinence, ?speech defect,

valvular HD history, confused, not eating. Patient had low albumin. What is the

cause of her functional decline?

a) end stage dementia

b) end stage renal

c) end stage liver

d) age related deliability

e) end stage stroke

43

Forensic Medicine

Direct cause of death

Female RTA, subdural hemorrhage after head fracture, herniation of the

brain, cardiac arrest, what is the cause of death Cardiac arrest,

Herniation, Head fracture, MVA

Forensic Case of an old man who had CHF, pneumonia, emphysema and then

cardiopulmonary arrest. What will be written as a cause of death in death

certificate Cardiopulmonary arrest

Patient with emphysema, heavy smoker , he developed pneumonia after

2 days he died, what is the cause of death Pneumonia, Cardiac arrest,

Emphyesima ,Heavy smoker

Underlying cause of death

Patient had car accident , multiple facture, cerebral edema then

cardiopulmonary arrest , what will you write in the death certificate , the

underlying cause of death:

a- Car accident

b- Fractures

c- Cerebral edema

d- Cardiopulmonary arrest

Man presents to ER with emphysema after road traffic accident. What is

the underlying cause of death Emphysema, Road traffic accident,

pulmonary embolism

Patient RTA, developed pneumothorax then uncal herniation and died.

Whats the underlying cause of death?

RTA

44

General surgery

Post operative care/ issues

Case of appendectomy 10 days ago, now has fever and diarrhea. What is

the most likely cause of his symptoms Enterocolitis, enteric fistula,

pelvic abscess

Patient presents 7 days post suppurative appendectomy with fever and

diarrhea. What is the likely cause ?

A) Rupture of appendiceal stump

B) Pelvic abscess

C) Fistula

patient 24 hours after surgery. Sob and fever. and crackles in the

basal lung?

*pulmonary embolism

*pneumonia

*atelectasis

The most common surgical complication within 24 hours is:

a. wound healing

b. myocardial infarction

c. incisional hernia

d. hemorrhage

e. wound infection

Plastic surgery

Burn with blue green infection which organism

Pseudomonas aeruginosa

30 years old male sustained burn in his anterior chest and abdomen

(second degree) his vitals are HR 100, BP 120/80, RR 18. After

clearing him from trauma team with no other fracturesm and

excluding other burns, height 160cm, weight 65 kg. How much fluid

to give in 24 hours.

a) 1.7 L/Day

b) 2.7 L/Day

c) 4.7

d) 5.7

[4 x weight (65kg) x burn % (18)]

45

Baby with scalding hot water burn. From umbilicus to groin and

both limbs affected. Child was in no distress. What will be most

affected?

* Sodium * Urea * Water balance * HB

DM patient , with leg pain ,blood sugar was fine, distal pulses present , foot

problem (red , hot, tender ) in the right side I guess , xray of leg , thigh showed

nothing ( xray provided) what is the next best step AKA

BKA

Symes amputation

IV antibiotics

Infections / ulcers

Woman with ?recurrent tonsillitis, had ulcer on the leg previously.

Now presented with another ulcer on the lower limb

what is true?

Patient with erysipelas, no hypertension, diabetes etc. Chronic ulcer on leg for 2

years. Comes with pain, tenderness, warmth. What is true?

a) Most common on face

b) Caused by staph

c) blood cultures is diagnostic

d) Penicillin is drug of choice Penicillin is drug of choice

Man with necrotized tissue in wound management

A.clean,remove necrotic tissue and leave open to be closed on later date

Case of a man developed fever, edema, crepitus in his leg, what is the most

causative organism

a. Clostridium perferinges b. strep. Pyogens c. staph aureus

Diabetic patient with pain from the knee to the inguinal ligament. Whole

area was swollen, red, warm with crepitation. Dx infection with strep

pyogens ( necrotizing fasciitis ), infection by strep, infection by staph

Middle aged man , present with one week duration of

swollen leg , red , hx of varicose veins , no history of DVT , he

being recently active . On examination small ulcer superior to

medial malleolus . Diagnosis ?

- DVT

- Cellulitis

46

- Neuropathic ulcer

Old lady bedredden , Both ulcers on same side on lateral malleolus and lateral

greater Trochanter

a. Decubitus ulcer

b. Venous ulcer

Patient with decubitus ulcer, which very deep with necrotic borders. What

is the best antibiotic ampicillin, erythromycin, cefotaxime,

clarithromycin, amoxicillin

Chest

Gs (PICTURE) . A smoker man developed neck mass swelling

which is irregular and moves with swallowing . What is X ray

showing ?

- Widened superior mediastinum

- Mid mediastinum widening

- Metastasis

Hernia

Case of incarcerated femoral hernia (laparotomy

Elderly female patient with abdominal pain & distension , tender mass in

the groin , increased bowel sounds Dx incarcerated femoral hernia

,Psoas abscess

Woman presented with acute abdominal pain. History of constipation.

O/E groin mass, increased bowel sounds.

- Paralytic ileus

- Incarcerated femoral hernia

Patient in inguinal hernia which of the following is indication for

incarceration Fever Constipation ,Vomiting ,Pain

Guy with inguinal mass appears while standing Indirect Hernia

old laborer reports swelling in the right groin. The non-painful swelling

is observable in the erect and not in the recumbent positions. Examination

reveals a non-tender reducible 4 cm mass in the right groin above and on

the medial side of the inguinal ligament. Which of the following is the

most likely diagnosis in this patient?

47

- Indirect inguinal hernia

- Femoral hernia

- Saphenous vein varicocoele

- Direct hernia

One patient has abdominal pain , vomiting and constipation for about 12 hours ,

past surgical history of laparoscopic umbilical hernia repair 18 months back ;

pneumoperitoneum, what could be the cause ?

o Amyand hernia

o Maydls hernia

o Small intestine adhesion

o Richter hernia

o Petersen hernia

Gs Truck loader come with a mass in standing position and goes away at

above and medial to inguinal ligament- ?

* Direct inguinal hernia

* indirect inguinal hernia

* Femoral hernia

*saphenous vein varicocele[painless,irreducible,below and medial]

General topics

A man who sustained some serious burn injury at home was intubated,

what other possible abnormality we should worry about Inhalation

injury, shock, sepsis

In summary patient had compartment syndrome after a car accident I think then

they were asking which is the sign in examination:

a. Pain out of proportion in examination

b. Pressure with compartment is 5mmhg a. Pain out of proportion in

examination

Mass at the mandible , hard and fixed and not mobile Parotid mass

with tongue protrusion what is Dx ? Parotid mass

Patient with fixed neck ma

move with tongue protrusion what is Dx Parotid enlargement, carotid

body tumor, thyroglossal cyst, ranula cyst

48

Most important information to tell the family of for prognosis about

the cancer?

* Size of tumor * Invasion * Histopathology *age

Patient was lost in the desert. His temp 42. And creatinine is very high.

Wat is the cause of high creatinine Dehydration, rhabdomyolysis

Which is unlikely to be a lateral neck mass

A) TB adenitis

B) Cystic hygroma

C) Branchial cyst

D) Carotid body tumour

Woman had ant bite to her finger and swollen finger. Then she

developed difficulty in breathing and expiratory wheeze but no

inspiratory stridor. Management?

- IV epinephrine

- IV salbutamol

- IV aminophylline

Patient has hematemesis, low BP, increase HR, next step IV fluids

49

Genetics

There was also cystic fibrosis question....lady who is pregnant came to know the

gender is female what are the chances of baby getting CF

o 100%

o 75%

o 50%

o 25%

cause of Downs syndrome Trisomy

Tall thin female patient with cardiac findings S4 murmur, arthralgia, headache,

what is the Dx Marfan

Waardenburg syndrome is a group of genetic conditions that can cause hearing

loss and changes in coloring (pigmentation) of the hair, skin, and eyes. ... People

with this condition often have very pale blue eyes or different colored eyes, such

as one blue eye and one brown eye Waardenburg syndrome

50

Gastroenterology

Esophagus

71 year old man with a 40-year history of cigarette smoking. Came with

progressive dysphagia from solids to liquid. Imaging showed a distal esophageal

stricture. What is the next best management? Esophageoscopy & biopsy

Do endoscopy to identify the reason

Endoscopy showed ulcer , what to give , Melena was present

o IV infusion pantoprazole

o H.Pylori eradication

question about patient with GERD and he is taking ranitidine. he has

positive breath test. what is his long-term management. answer is antibiotic

(because of Hpylori. PPI was not in the options

A 60 year old man was diagnosing with cancer in the mid third of the esophagus.

What is the sign that makes this patient most likely untreatable:

Hoarseness of voice

cancer of mid 1/3 of esophagus . no history of smoking diagnosis ?

* adenomatous adenocarcinoma *squamous cell carcinoma *transitional

carcinoma

Carcinoma of middle third of esophagus. Most likely?

- Squamous cell carcinoma (More common in upper and middle

third)

- Adenocarcinoma (More common in lower third of esophagus)

- Transitional cell carcinoma

Patient with dysphagia, nausea, vomiting. Narrow lower 1/3 of esophagus and

dilated esophagus on CXR. Diagnosis:

a) Scleroderma

b) GERD

c) Achalasia***

D) diffuse esophageal spasm

Women who was experiencing dysphagia. Manometry done showed increased

Lower Esophageal Sphincter tone.

a) Achalasia

Achalasia cardia

Case of UGI bleeding, endoscopy shows 1cm ulcer with a non-bleeding ulcer

and a visible vessel was seen . What to do next?

a) Injection of sth in the vessel

b) hat is it Clipping the vessel?

Endoscopy for dysphagia for solids

51

Non bleeding ulcer on endoscopy:

a. clipping

b.

old man with progressive dysphasia for liquids then solid now

he can only take liquids , Dx? Oesophageal cancer

old man with progressive dysphasia, now he can only take liquids what

do you want to do ?

A) endoscopy

B) barium

C) CT

Old patient with a history suggestive of esophageal cancer what is the initial

investigation?

A man has dysphagia for solid food and lost 15 kg what is the next appropriate

next step

a. Barium study***

b. Endoscopy

Patient found to have stricture on upper GI series , he has progressive

dysphagia , do you do dilation first or endoscopy with biopsy first

Patient with esophageal cancer, what makes dx of cancer untreatable in

him Cannot speak full sentence, increase RR 50, po2 <90, PCO2 >45

What makes esophageal cancer untreatable recurrent laryngeal nerve

involvement, Involvement of para carnial lymph nodes

Young female, dry cough, increases in the morning, clear throat and

voice hoarseness for 6 weeks, no fever GERD, Allergic

rhinosinusitis

Stomach

Male known case of GERD , takes ranitidine 150 mg , not

relieved and is being awaked at night because of pain . No weight

loss or red flags , what to do ? (repeated from 28 Jan)

- Investigate for H. pylori

- Double dose of PPI

- Endoscopy

- Give triple therapy

Patient with GERD and he is taking ranitidine, he has positive breath test

antibiotic (because of H. pylori)

Succusion splash

Gastric outlet obstruction

Case of a guy with GIT symptoms. On auscultation of the abdomen a

succession splash was heard. What is most likely Diagnosis Gastric

52

outlet obstruction, colonic obstruction, intestinal obstruction

Man with non-bilious vomiting O/E there is? succusion splash what no

bowel sounds is the Dx gastric outlet obstruction, small bowel obstruction,

colon obstruction

72 years old lady with osteoporosis of both knees for which she take

abdominal examination, bowel sound were absent & findings

suggestive of peritonitis. Regular HR of 105, BP 135/90, RR 19.

Amylase & lipase were normal. What is the diagnosis

a) Perforated empyema of gallbladder

b) Acute pancreatitis

c) Mesentric venous thrombosis

d) Perforated peptic ulcer

e) Perforated cancer of transverse colon

A patient K/c of peptic ulcer. Was given ranitidine and at night he woke

up due to epigastric pain, what is next line of Mx Abdominal/CXR

History of PUD , presented with rigidity and rebound abdomen Perforated small

intestine

Lady / man with NSAID intake then develop epigastric pain PUD

Gastric cancer , what substance will be increased ? with gastric polyps

o Histamine

o Gastrin

o Somatostatin

o CCK

Patient with gastric fundus polyp ..which of the following will be high

a. Gastrin ***

b. Cholecystokinin

Male known case pf peptic ulcer , presented of acute epigastric

pain and abdominal rigidity . What to do ? (from 28 Jan)

- Erect chest X-ray

Patient with benign ulcers in endoscopies what medication to give which can

provide fastest albeit briefest control

o Antacid

o H2 blocker

o Proprenthaline

o Belladonna alkaloids

mass Esophageal cancer, gastric cancer, pancreatic cancer

Intestine

53

Child presented with fever, periumbilical pain initially, then localizes to

the right lower quadrant pain, examination showed tenderness and

rebound tenderness in the RLQ area, what is the

diagnosis Appendicitis, Mesenteric adenitis

Abdominal pain shift from umblicus to RIF+nausea vomiting, No

rebound tenderness, PR sever tenderness ,Dx ? appendicitis, lymphatic

adenitits, divrticulitis

First symptom of appendicitis in a patient who had generalized abdominal pain

then localized to RLQ

a. Generalized abdominal pain***

b. Fever

c. Nausea

d. RLQ pain

Child presented with central abdominal pain that radiated to RLQ

, nauseated but no vomiting , rebound is positive . What is the most

likely diagnosis ?

- Mesenteric adenitis

- Appendicitis

Case of pelvic brim appendicitis, diverticulitis

Patient presented with pain and tenderness in RIF (?examination signs were

positive), one day prior to this presentation he presented to the ER with

generalized abdominal pain, what to next?

a) cbc and urinalysis

b) cbc and urinalysis and ultrasound of abdomen***

c) surgery for appendicitis

Old man with fever 39.8 , known case of HTN and other co-morbidities,

rebound positive, RLQ fluid collection 4 cm ?

a) Laparotomy drainage of the abscess

b) Aspiration of abscess , radiologically guided

c) Laparoscopic appendectomy

case of voluminous stool and stool reducing substance +

c. Glucose-galactose malabsorption

Child admitted with 10% dehydration due to diarrhea. Low grade fever (38oC).

ne for 12 hrs. What Ix will you order?

Urea & electrolytes

child dehydrated and he is 9kg how you will calculate the NS u want to

give 10% x Body weight ( in grams

54

intussepsion

What is the cause for intersussption

Actually it was a long question but this is the summary Viral infection that

causes lymph node enlargement

Clostridium deff outbreak in the hospital. What measure to take?

Use alcohol gel before contacting the patient

constipation and air fluid level on Xray management ?

intestinal obstruction

Woman presents with lower abdominal pain, nausea and vomiting for

2 days. She has not moved her bowel for the past 7 days. On examination,

no tenderness, bowel sounds? Multiple air fluid levels.

-Volvulus

-Intussusception

-Subacute intestinal obstruction

-Paralytic ileus

Constipation and air fluid level on X-ray management SBO

management Laparotomy

Diabetic patient with abdominal pain, severe, constant with distension,

not tender and history of AF. Diagnosis Mesenteric artery occlusion,

acute pancreatitis, sigmoid volvulus

Diffuse abdominal pain-

first sign of appendicitis

family ate meat pie last night and all have vomiting. best next step.

give IV saline

Lady presented because of vomiting for one hour as well as her

husband and son after eating a left over meat pie from a day before

. What is best next step? (repeated from 28 Jan)

- IV fluid

- IM Ranitidine

- Oral magnesium hydroxide

- IV ampicillin

Mariam is 30 month old child her mother brought her to the clinic

because she a long history of diarrhea mother said " she never had a

form stool" she has non bloody non foul smell diarrhea which had 4-6

times a day. Mariam is delightful child with a normal growth for her

age. What is the diagnosis?!

a) celiac disease

c) disharridaase deficiency

55

Lady presented with history of abdominal pain , relived by defecation , scope

done and it showed normal mucosa IBS

Patient being treated for oat cell carcinoma loosing weight, no appetite cause of

chemotherapy, nausea and vomiting and low appetite even when wife cook

favorite food..

a) prochlorperazine

b) Amitryptilline

c) Megesterol acetate

Megesterol acetate

Patient noted to have volvulus in sigmoid colon with other volvulus from the

small intestine around the first one what do we call this volvulus

o Compound

o Double

o Knotted

o Combined

Patient with abdominal pain and vomiting with CNS symptoms (seizures) , what

is the organism Botulism toxin

Patient with central abdominal pain , radiating to the right side of the abdomen ,

with pain in DRE , no raise in WBCs or Neurotrophilia , no rebound o

Mesenteric lymphadenitis

o Appendicitis

o Renal colic

o Diverticulitis

25 years old female, presented with 3 months history of fever, pain in the

RIF, on examination, mass was found in the RIF, abd x-ray showed

calcifications in the RIF, dx crohns disease, appendicitis, TB, right

sided diverticulitis, appendicular mass

Young lady known case of peptic ulcer which she was treated for 2

years ago. ( history of alcohol intake) 4 hours ago developed abdominal

t moved with respiration no bowel

sound , epigastric tenderness.no rebound tenderness what is diagnosis ?

* Acute intestinal hemorrhage

* small Intestinal perforation

* Intestinal obstruction

Patient with PUD , now positive rebound in the epigastrium o Perforated

Small bowel

Ulcer increased with hunger and partially reduced with food. Weight gain.

Location?

56

a) body of stomach

b) first half of duodenum***

c) jejunum

Note: Food aggravates gastric ulcer and relieves pain in duodenal ulcer.

Colon

old man with painless bleeding and colonoscopy revealed multiple diverticula

a. Dietary modification

b. Nd yag laser something

60 year lady, LLQ pain, fever for several weeks, developed

generalized abdominal pain and tenderness, guarding, rigidity,, no

bowel sounds perforated diverticulitis

diverticulitis

52 years male with heavily per-rectum bleeding. 4 days ago he had

colicky abdominal pain. His HR 140, RR 22, and low BP. He was

transfused with RBC pack. What is the best diagnosis?

a) Colon cancer

b) Rectal cancer

c) Diverticulor

Patient with diverticulae and bleeding colonoscopy showed serpanguinous

vessels in cecum. What will you do?

a) Diet

b) Laser

c) Tagged RBC

Diet

37 yrs old patient was found to have multiple colonic polyp and he removed it

surgically ( picture provided ). His father and brother had colon cancer. what

should he have done before it

a. Annual colonoscopy

b. Colectomy at one year old

c. Occult stool blood annually colonoscopy annually

d. Sigmoidoscopy annually

An old patient presents with diarrhea and distended bowel loops. Not

tender, no guarding.

A) Ascending colon CA

B) bowel obstruction

C) Descending colon CA

60 years old man presented with 2 weeks history of intermittent

abdominal pain and diarrhea. Abdomen is distended with hyperactive bowel

sounds , Xray of abdomen shows gas in small and large intestine . On

57

examination rectum is empty . What is the most likely cause ?

- Cancer in descending colon

- Paralytic ileus

- Cancer is ascending colon

- Diverticulitis

Male Patient with diarrhea. And exaggerated bowel sounds and

abdominal distention. OE empty rectum. What is the DX? Descending colon

cancer

A man with empty rectum, hyper active bowel sounds and diarrhea with

abdominal pain. cancer in the bowel (descending colon)

Most important piece ( to tell the family) of information for prognosis

Size of tumor, Invasion, Histopathology, age

Old man had colon cancer , what most important point to mention

to family regarding prognosis ?

- Depth of invasion of tumor

- Location of tumor

- Metastasis ???

A 23 years old male patient has developed perianal fistula and

constriction in the ilium. The underlying cause of his condition is:

a. Ulcerative colitis c. Inflammatory bowel disease

Non caseating granuloma

Crohn's histopathology

a. Non caseating granuloma***

b. Caseating granuloma

c. Crypt abscess

d. Thinning of the affected wall

A Patient with tenesmus and diarrhea and rectal bleeding bright red blood.

a. Diverticular disease

b. Rectal cancer

c. UC

Rectal CA

Child have many polyps in the colon what is the gene ? APC

Gross picture for bowel with multiple polyps what is the gene mutation

responsible for this disorder? APC

colonic cancer 1st symptom:

a. weight loss

b. anal pain

c. abdominal pain

58

d. bleeding ( change in bowel habits will be the first symptoms )

Hx of colectomy, presented with itching, imaging of biliary tract showed

narrowing of intrahepatic bile ducts. periductal onion skin fibrosis in liver

biopsy. What is the underlying cause?

a) primary biliary cirrhosis

b) ulcerative colitis***

C) schistosomiasis

D) ulcerative colitis

Treatment of Ulcerative colitis sulfasalazine and mesalazine,

Corticosteroids.

Male Patient with diarrhea. And exaggerated bowel sounds and

CRC, descending

Patient come with polyps in the colon she did polyp andenoma restrictions few

years ago , her brother died at age 40 from colon cancer , she have history of

ovarian cancer, what is the diagnosis?

a. FAP

b. HNPCC**

Type of colon cancer ..patient presented with family history of father 50 years

old diagnosed with colon cancer 10 years ago and her brother went into colon

resection procedure what is the type of cancer

a. Peutz jeghers syndrome

b. Hereditary nonpolyposis colorectal cancer***

c. Familial adenomatous polyposis

The lady had ovarian cancer hx, colon biopsy showing colorectal carcinoma:

a. HNPCC**

b. FAP

Diarrhea after receiving amoxicillin Metronidazole

Anal and rectal diseases

Git young man with painless bleeding -haemorrhoids

young man with painless bleeding . Fresh blood on tissue while he has normal

bowel movement and diet. Dx? Hemorrhoids

18 years old male presented with painless bleeding since 2

weeks , fresh blood seen on tissue papers , no change in bowel

habit . No other symptoms . Refused rectal examination . Diagnosis ?

- Hemorrhoids

- Anal fissure

- Rectal hematoma

- Fistula in ano

59

Anal cancer drain to which lymph nodes ?

- Inguinal

Patient with rectal cancer, what is the site of lymphatic metastasis

Perivesicular, Inguinal, Paraoartic

Case of rectal cancer. How do we know the severity ? Depth of invasion

Old man came with fresh bleeding PR. O/E: Tachycardia,

hypotension. Next step?

- IV fluids

- Colonoscopy

Old man rectal bleeding, weight loss, muscle guarding diagnosis

Colon CA

Patient with per rectum bleeding, diarrhea, tenesmus, what is the

diagnosis? Rectal cancer, Diverticulosis

Patient came with rectal bleeding and fever? Which investigation will give the

definitive diagnosis ?

a. Colonoscopy

b. Occult test for blood

c. Sigmoidoscopy

57 years man, diarrhea with blood & mucous , next step Do PR

exam and prepare for colonoscopy

Liver

Alcoholic patient diagnosed and was under treatment for pneumonia died .on

autopsy the hepatocyte showed macrovesicular lipid deposit

a) HCC

b) Chronic alcoholism **

Patient has liver problem (scenario??? ) his platelet is 10000

. Why it is contraindicated to take a liver biopsy ?

- Low platelet

platelet <60000, elevated PT and PTT

or INR>1.6, or hemangioma

CT scan with a clear hepatic contusion (which is the answer

Hepatic contusion

Indian man came recently from India , had jaundice with pale color stool , labs

was provided with increase total bilirubin level and low indirect what is the

cause ? Obstructive jaundice ( obstructive cause)

Prehepatic hemolysis

Man with painless jaundice, smooth globular mass in the RQ. Dx?

a) Pancreas cancer Pseudo cyst

60

Man died because of pneumonia , autopsy of liver showed multiple masses ,

during his life he had a consistent low Hb what is the cause of this autopsy

picture ?

o HCC

o CRC metastatic

o Hydatid cyst

Patient has symptoms of UC on medications, developed jaundice and

abnormal LFT Primary sclerosing cholangitis ( Tx:Urodeoxycholic acid )

Patient has symptoms of UC on medications, developed jaundice and abnormal

LFT. Dx? Sclerosis cholangitis

Patient alcoholic for many years . On autopsy, liver was more than 2 kg (normal

for his weight is 1.8 kg), and large cholesterol droplets with areas of necrosis.

What is the underlying cause ?

a. Chronic alcoholism***

b. acute hepatitis c

c. Biliary lithiasis

d. Hepatocellular carcinoma

61 years old, hx of liver cirrhosis due to hep C, abdomen distended,

examination revealed ascites (transudate) fluid WBC is 150, what is the

next step of management lactulose therapy with protein restriction,

peritoneal lavage, start interferon therapy, give antibiotics according to

the result of fluid culture

HbsAg ve, HbsAb +ve, HbcAb ve, HCV Ab ve Patient is

immunized against HBV

Weight loss 8 kg, Liver biopsy showed patchy necrosis, ballooning degeneration,

PNML infiltrate

a. Alcohol cirrhosis

b. Alcoholic hepatitis

c. Alcoholic siderosis

d. Alcoholic fatty liver***

e. HCC

Gall bladder

42 years old lady , has 5 children , takes an OCPs . Developed an

upper right quadrant pain, nausea , vomiting , noticed yellowish

discoloration of eyes and dark urine . Labs show high ALP and

AST . What is her most likely diagnosis ?

- Biliary colic???

- Drug induced cholestasis

61

female with acute severe abdominal pain. she uses OCP and has

jaundice. answer is OCP induced gallstones. DONT be fooled by biliary

colic because it does not cause jaundice

lady using ocp came with jaundice proceeded with RUQ pain one day

ago , high ALT, ALP and bilirubin, what's the diagnosis ?:

A) biliary colic

B) drug induced cholestasis

Pruritis and jaundice, dx cholestatsis primary biliray chirrhosis

Hepatosplenomegaly. old man with normal AST/ALT but increased

ALP. Primary biliary cirrhosis

Case of liver enzymes normal except mild elevated ALP, whats your

Dx Primay biliary cirrhosis, alcoholic cirrhosis, viral hepatitis

Woman presents with right upper quadrant pain, pruritus and jaundice.

ANA was negative but AMA was positive. LFTs deranged with high

ALP. Mx?

Ursodeoxycholic acid (This is a case of primary biliary cirrhosis)

Primary biliary cirrhosis: high ALP, AMA +ve, hepatosplenomegaly

treatment: urodeoxycholic acid, definitive treatment is Liver Transplant

Patient had Primary biliary cirrhosis what to give her > o USDA

Case of biliary cholecystitis

Charcot trial criteria what is it? RUQ pain + fever + jaundice

Patient presented with gall stone and the ultrasound show that common bile duct

) so

what is the next best step

a. ERCP

b. MRCP if asymptomatic and no stones in the CBD then expectant

management.

if symptomatic then elective cholecystectomy.

if there are stones in the CBD, then antibiotics and ERCP if patient is stable, if

not the PTC

54 years old female, C/O abdominal pain after eating, mostly in RUQ,

sometimes colicky, she reduced her meal but pain not reduced, next

step US for RUQ abdomen

Patient went in for some kind of biliary tract surgery. She developed post op

jaundice. No pain. What is the cause of postop jaundice? Clipping of CBD

Woman with RUQ pain radiating to the back. Worsens on lying

positive. Dx?

- Cholecystitis

62

- Gall stones

- No pancreatic options were mentioned

What is the cause of postop. Jaundice Clipping of common bile duct

,Cholangitis ,Pancreatitis ,Drug induced liver injury

Pancreas

A marker for poor prognosis of pancreatitis in the first 48 hours LDH level

800

prediction of poor prognostic factor of pancreatitis

LDH > 350, AST > 250 )

B>incresed amylase

C.incresed triglycerides

acute pancreatitis

epigastric pain radiating to the back with history of alcoholic intake

acute pancreatitis

Presentation of typical acute pancreatitis. Patient with history of

peptic ulcer, and now is coming with rigid abdomen guard

test will confirm the diagnosis? erect x-ray ,Abdomen xray, Abdomen US

,GASTROGRAPHIN with fluoroscopy

Lady presented with biliary pancreatitis , treated by IV fluids and antibiotics was

given , now she have cholecystitis , USG showed multiple stones in CBD and

single stone in the gall bladder , what is the next step ? o ERCP

o MRCP

o CT abdomen

o Cholecystectomy

male patient 60 year old alcoholic with abdominal pain weight loss and

jaundice, CA of the head of the pancreas

Old man has epigastric pain radiating to the back, alcoholic and smoker

for the past 20 years. DX CA head of pancreas

Old man, with jaundice and weight loss , mass in the epigastrium ,what is the

cause o Cancer head of pancreas

Middle age male , with epigastric pain radiates to back more at night ,

continuous jaundice, weight loss, epicgastric tenderness, abdominal mass

, HGB low , ALP high , Transaminases mildly elevated . US shown

(dilated biliary tree probably) Head of pancreas tumor , Colon cancer

PAINLESS progressive jaundice with weight loss and smooth globular mass in

the epigastrium Pancreatic adenocarcinoma

63

MRCP

a. amylase + pancreatitis and gallstone

Old man typical presentation of pancreatic cancer , what is

the most likely finding ?

- Steatorrhea

- Vomiting

weight loss+clay coloured stools+epigastric pain radiating to the back

A.Ca pancreas B.gastric CA

Patient had acute pancreatitis, he was alcoholic, Admitted for 10 days to

treat pancreatitis, now came again with fever and chills and tender

epigastrium Pancreatic abscess, duodenum ulcer

patient present rt hypochondrial pain and nausea and vomiting, no aggravating or

relieving factor.no jaundice. investigation?

A.Lipase

B.Bilirubin

C.uric acid

D.LDH

Others

55 years old heavy smoker (40 cigarette) per day for long

time + alcohol abuse . He presented with left jugulogastic

lymphadenopathy , what cancer he is likely to have ?

- Oropharyngeal

- Liver

- Lung

Patient with hematemesis and tachycardia and low blood

pressure like 70/50 , what to do ?

- IV fluids

- Upper GI endoscopy

1 hour vomiting .normal BP .Normal pulse and dry tongue IV fluid,

IM ranitidine

Patient with hematemesis , endoscopy showed artery not bleeding what to do

o Clipping

alcholic patient was advised to quit drinking. he came back 1 week later with Hb

10, Retics 5% and hight cholesterol 300

s syndrome***

b) Alcoholic siderosis

c) Budd chiari syndrome

d) Hepatocellular carcinoma

64

Mechanism of omeprazole inhibition of the H+/K+-ATPase system

Patient with ascites. Tapping done showed 350 neutrophils. what will you give

him?

a) 3rd generation cephalosporin

b) Other options

CT showing tumor , normal ALT,AST , source of the tumor Liver,

Pancreas, Epigastrium, Biliary duct

Woman come to dubai last week came complaining of vomiting 2d ago ,

Alcoholic for 20 years she admitted to drinking 2 glasses of beer every night

Which of the following is true?

a. AST>ALT ratio 2:1**

b. Reducing vit k

c. Reducing y_glutamyltransferase

d. Reducing alkaline phosphatase

e. Micro hypochromic anemia

Smoker man with painless lower lip ulcer . Dx Squamous cell

carcinoma, Herpes

65

Hematology

RBCs disorders

26 years old female, presented with recent history of tiredness, Hb was 8

with reticolcytosis, blood smear shows numerous micro-spherocytosis,

haptoglobin was low, comb test positive, dx autoimmune hemolytic

anemia

normal reticulocyte, normal Hg, normal WBC, normal platelets. What her

diagnosis B- thalasemia trait, Sickle cell trait, Hereditary

spherocytosis, G6PD

Child presented with pain in the hands blood film showed sickled cells

Sickle cell anemia

Child with painful hands and feet, the spleen was 2 cm palpable and tender.

What does he have?

a. SC

b. SS

c. Thalassemia major

d. Thalassemia minor

Emirati guy want to travel to India , he revived some medication to prevent him

from getting malaria , he became weak , jaundiced ... what is the possible cause ?

G6PD deficiency

What is the percentage of GP6D In the offspring 25%

Clinical scenario and a picture of a peripheral blood smear showing

fragmented RBC (schistocytes, helmet cells), it also had what looked like

sickle cell but SS anemia was not a the options, what is your Dx

Microangiopathic hemolysis, Beta thalassemia, G6PD, spherocytosis

One blood film showing hemolytic anemia. G6PD High

retics/schitocytes ( methyldopa drug induced hemolytic anemia )

Photo for blood film showing bite cells. Some large cells with multiple

nuclei . In scenario woman with fatigue . Taking methyldopa for essential

hypertension. What's causes for her anemia Spherocytosis ,

Microangiopathic anemia ,Drug induced hemolytic anemia

Middle aged lady with excertional dyspnea on climbing stairs, relieved by

rest, and symptoms recur when she

CHF, IDA, CAD

44 years old lady presented complaining of SOB , dizziness

& exertion on walking or climbing stairs but no chest pain . She

66

is known diabetic and hypertensive complaint to medications .

Her weight is 81 kg and height is 1.66 cm . No chest pain , eats a

balanced diet . No history of coronary arterial disease , no

features of hypothyroid . Physical examination shows a normal

heart clear lung with no crackles . Most likely cause of dyspnea ?

- IDA (iron deficiency anemia)

- CAD ( coronary arterial disease)

- CHF (congestive heart failure)

- Hypothyroidism

Case of IDA, low Hb, low MCV. What will you find? Low ferritin

after 3 months

Patient had iron deficiency, received iron therapy , for how long should he

continue the treatment 3 months after reaching the targeted Hb

Boy with low mcv and high RDW (18%) and normal reticylocyte count

b) IDA***

c) Thalassemia trait

d) Hemolytic anemia

MCV <80

Diagnosis of IDA. *low ferritin *low tibc

An 8 years old has a history of fatigue, her Hb level 8, the most like

indicator of hemolysis would be:

a. Direct coomb's test

b. Indirect coomb's test

c. G6PD level

d. Haptoglobin level (will be low)

Girl with iron deficiency anemia which one of the following related to

this condition?

* pin worm

* Pig tapeworm

* hook worm

* Fish tapeworm

Blood group of the fetus of mother O negative and had antiglobulin positive,

second pregnancy, father blood group not given, delivered her baby and later on

after the baby started to have features of hemolysis (hyperbilirubinemia, anemia,

hypoproteinemia).

What is the blood type of the baby

a. B positive

b. A positive

67

c. AB negative

d. O positive ***

e. A negative

31 year old with weakness, mcv and TIBC low. S. iron high, basophilic stippling

and target cell seen-

a) sideroblastic

70 years old make came with complain of weight loss BMI was 18,

you did upper and lower endoscopy, blood tests and other investigations all

we're normal, you instructed him to come after 4 weeks with a diary, he

came back he lost more weight and couldn't produce the diary ?

A) Repeat blood test ( I think CBC)

B) Repeat endoscopy

C) Repeat chest x Ray

D) Repeat echo ??

E) Cognition evaluation

F) Repeat colonoscopy

patient on methyldopa presents with anemia. And a third degree

systolic murmur. Hb 8, PLT 87, 000. Diagnosis? Blood picture given

(exactly the same)

A) Drug induced hemolytic anemia

B) Microangiopathic hemolytic anemia

(methyldopa drug induced hemolytic

anemia

Anemic 25 year old Low RDW, low MCV, splenomegaly, high iron, basophilic

stippling. What is it true about it?

a. Target cells are seen in the film

b. It is a congenital disease b

Basophilic stippling on blood smear, decreased mcv, increased iron level,

splenomegaly and the pt is anemic, what is true about the pt condition ?

a. Target cells are seen in the film

b. It is usually congenital disease a

I remember there was a similar question of woman with low MCV and

basophilic stippling and enlarged spleen what's the diagnosis

a. Thalassemia trait

b. Sideroblastic anemia look at the Iron

If high > sideroblastic

If normal > thalassemia trait

68

Basophilic stippling on blood smear, decreased mcv, increased iron level,

splenomegaly and the pt is anemic, what is true about the pt condition ?

a. Target cells seen in blood film

b. It is congenital

c. Give iron supplements

d. RDW will be decreased

e. Can't remember other options i think the right answer is target cells are

seen.

RDW usually is increased

it can be congenital or acquired

Boy with mildly hemolytic anemia and intermittent Jaundice He has right upper

abdominal pain , management?

Patient with features of Sickle cell? What supplement must he take?

a. Folic acid***

b. Riboflavin

c. cobalamin

Mode of inheritance of thalassemia.

- Autosomal dominant

- Autosomal recessive

- X-linked

Platelets / coagulation disorders

4hrs post gastrectomy bleeding from venipuncture sites

1. low platelet count

b. decreased/increased antifibrinolytic agents

low platelet count

Cause of bleeding 4 hrs ? post gastrectomy from venepunture sites,

pupuric lesion on lower limb, dressing over the surgical wound was

soaked

A.low vitamin k levels

B.ineffective electrocautery

C.low platelet

D.low anti fibrinolytics

Post operative , oozing from the surgical sites and venupuncture what is the

expected lab finding Low platelets

12 years old female presented with hemoarthrosis, bruises on her body.

Her grandfather has hemophilia A, what make you exclude hemophilia A

from the history Female gender, Present of bruises, Grandfather had

hemophilia A

69

One kid with hemophilia/B?, The gene for hemophilia is carried on the

X chromosome.

case about hemophilia. What is the diagnosis Factor VIII

deficiency, Factor X deficiency

Boy with bleeding problem, grandfather has same

bleeding Hemophilia A

Small boy, with knee swelling and several bruises on legs. BH/o bleeding for 2

days after circumcision. Grandfather with history of disease that causes

prolonged bleeding. Dx? Hemophilia A

Boy presented with knee effusion and warmth >> Typical case of

hemophilia , positive family hx from grandfather side ,What's the clotting

factor deficient in hemophilia A 9, 8

patient has nose bleed recurrent epistaxis and headache , Upon exertion and

headache and he has lower limb pain after exercise and weak peripheral pulses as

well, and the choices included :

o CT chest

o check for factor 8 and 9

o Xray

o Echo

o ECG

12 year old girl had an untraumatized nose bleed. She had the same

symptoms sometime back. Her 6 years old brother had a prolonged

bleeding after a tooth extraction. If you suspect an inherited disorder,

what is the method of inheritance:

a. X linked

b. Autosomal dominant (Von willebrands disease, type 1 or 2)

c. Autosomal recessive

Mixed Cryoglobinemia

VTE

48 years man present to hospital with sob and rt chest pain. His vitals

normal except tachycardia. CXR normal. His po2 is 60, his co2 is 35 and ph:

7,44. After given his o2 mask his po2 improve to 85 and other agp where the

same. What is ur next step in management CT angiogram, lung function

test, doupler us

70-year-old lady had a hip fracture following which she underwent a

hip replacement. 7 days post operatively she developed symptoms of

breathlessness, chest pain? (PE picture) when ambulating with her

physiotherapist. What is the best test to be done?

- Lower limb USS Doppler

70

- V-Q scan

- CT angiogram

- X-ray chest

Patent had hip surgery, developed signs of PE. What is the investigation you do

to lead to Dx? CT Angiogram

A patient, post LSCS developed shortness of breath. What is the source of her

DVT? Lower limbs

family hx of DVT

a. Factor V leiden def

Lady on OCPs comes with sudden onset of chest pain

After a long haul flight. What caused her symptoms? Travel History

was mobilized for first time after

admission and then developed sudden unilateral chest pain that has resolved

a. Pulmonary embolism**

b. Bronchiectasis

c. atelectasis

Lady presented with symptoms of DVT , she had family history of HTN , factor

sth deficiency and she is on OCP , which one is not a contributing factor to her

DVT Hypertension

Post op pleuritic chest pain 10 days-

o Pulm embolism

Oncology

Patient with low back pain, anemic, increase Ca, increase total protein,

next step bone marrow biopsy (multiple myeloma, 10% plasma

cells)

Scenario of leukemia in a 72 year old with tear drop cells on bone

marrow biopsy and some RBC were nucleated,Dx myelofibrosis

,CML, AML

Increase BUN, increase creatinine, ALP and AST normal, tear drop cells,

diagnosis AML, Myelofibrosis, CML, ALL

Bone marrow biopsy showing tear drop cells. Dx? Meylofibrosis

50 something years old patient with fatigue, early satiety. Labs show tear drop

cells. Bone marrow shows fibrosis with hyper cellularity. JAK 2 postive,

BCR/ABL negative.how will you manage?

a) Thalidomide

b) Ruxolitinib (Ruxolitinib JAK2 inhibitor)

c) Splenic radiation

d) colchicine

71

The patient with investigation showing high IgG, high creatinine,and anemia of

chronic dz, (Monoclonal gammopathy), What symptoms do you expect he would

have?

a. Bone pain

b. Hepatomegaly

c. Splenomegaly

Patient with hepatosplenomegaly, weight loss, blood film showed

immature granulocyte in different stage of maturation, decrease alkaline

phosphatase CML, CLL, Leucomoid reaction, ALL

patient had anemia , thrombocytopenia, immature granulocytes at

different stages of maturation. hepatosplenomegaly, night sweats, wt

loss. lap was decreased ?

* CML *CLL * ALL * AML

Elderly lady came with splenomegally , HBG 9 , platelets : ,

LAP (leucocyte alkaline phosphatase) is low . Blood smear shows

atypical granulocytes at different stages of maturation . What is

the most likely diagnosis ? ( From 28 Jan)

- CML

- Leucomoid reaction

- ALL

- CLL

Patient has AML, how to determine the severity cytogenic

karyotyping, immunophenotyping

Young boy, recurrent chest infection, pale on examination, has petechia,

diagnosis lymphocytic leukemia, mixed immune deficiency

47 year old with clinical symptoms of CML with CD20 +ve, Mature

Lymphocytes were 40% in BM. No active complaints now. What is the most

likely complication that he will develop later?

1. Disease progressing to acute leukemia***

2. Serious infection

12 years old boy , presented with increase fatiquability over a period of time ,

with weight loss and enlargend cervical lymph nodes , CXR was done and

showed widened mediastinum , wbc 20 mainly lymphocytes , rbc 2 , plt 20.000 ,

blood smear shows abnormal blast cells what to do for diagnosis Bone

marrow

Lymph node biopsy

Mediastinal biopsy

72

CT scan

will you do for staging US of abdomen and pelvis, biopsy of liver and

spleen, CT scan of abdomen and pelvis

ALL diagnosd. Started on chemo(vincristine). Had a seizure. treated with

diazepam and now in ICU. They ha sent U/E before. What electrolyte are u

expecting. Urine osmolality is 900.

a) Hypo nat+hypo k

b) Hyper NA & hypo K

c) Hyer k + hypo mag

d) Hypouricemia+-___

e) Hyper k+hype__

answer If they mention hyperuricemia and hyponatremia is the best answer

if not then its A

child with leukemia. started on chemo. very low neutrophils. has

fever. no focal signs of infection. management.?

* do culture and wait for results

*start 3rd generation antibiotics

*give immunoglobulin

Patient with leukemia was started on chemotherapy. He developed

fever and labs showed severe neutropenia. There were no signs of the

focus of fever. Mx?

- Do culture and wait for results to start antibiotics

- Start 3rd generation cephalosporins

- Give immunoglobulin

Others

blood transfusion reaction

Patient presented with ulcer on medial malleolus ,

hyperpigmentation , mild swollen calf , varicose veins , what to do?

- Compression bandage

- 80 mg enoxaparin

73

Infectious Diseases

Respiratory infections

2 yr old kid with cough and fever. high neutrophil count. x-ray

showed infiltrates in one lobe. most likely cause?

* viral pneumonia * strep pneumonia

Best diagnostic test for tb ZN stain in bronchial wash

Indian patient had cough, weight loss, night sweat, CXR: lower zone

infiltration, how to confirm Esophageal wash with Ziehl Neelsen

stain (the acid-fast stain)

CASE OF pulmonary lymphadenitis

Case of pulmonary Tb

Child traveled to his home country india , exposed to grandfather with Tb ,

asymptomatic now , did PPD positive , what is the next step ?

a) Bronchial / sputum culture for AFB

b) Gastric aspirate for AFB

c) Chest xray Gastric aspirate for acid fast bacilli

Indian guy, hosp Porter, low grade fever, night sweats, cough weight loss,

x-ray described. Diagnosis pulmonary TB

man with fever cervical lymphadenitis for 3 weeks, histology reveals necrotizing

granulomatous inflammation but mycobacterium tb immunostain something is

negative

a. tb is excluded

b. can't be excluded have to do further tests

Indian patient presented for cough, fever & and night sweats for

3 weeks . Probable diagnosis?

- Tuebrculsosis

Patient with cervical lymphadenopathy and feeling tired. Biopsy was done which

showed necrotising granulomas

mycobacteria stain was done which was negative

which of the following are true?

a) Tb has been excluded

b) this is a bacterial infection

c) cannot exclude Tb, need more investigations

d) it is due to diminished blood supply to the lymph nodes

cannot exclude Tb, need more investigations

74

Elderly male had a community acquired pneumonia , culture- streptococcus,

started antibiotics; situation worsened.

cavitation on x ray

what was the organism? o legionella

o Chlamydia

o Pseudomonas

o Staph aureus

Ethopian lady works as housemaid , had dry cough , did CXR normal ,

quantiferon + , no night sweats or weight loss Repeat CXR in 3 months

Isolate and contact IDU

Repeat Quantiferon in 3 months

Start treatment INH for 9 months

Reassurance

if u will give Isoniazid for treatment of TB, which vitamins u should

supply vitamin B6 (pyridoxine)

Measles transmission droplets mainly (direct) or airborne

Most likely organism to cause infection post splenectomy Klebsiella,

E.coli, Streptococcus pneumonia

Patient with fever, night sweats, matted LNs. Dx? Tb lymphadenitis

Patient with productive cough , yellowish sputum , fever for 3 weeks , now

become blood tinged , weight loss 2 kgs o TB

o Malignancy

Scenario of measles. Ques was what will help in mortality or morbidity?

O Vit A

o Vit C

o Rabivirn

o IG

Patient, Hx of heroin intake for long term, he will die because of

cerebellar atrophy, pontine hemorrhage, peripheral neuropathy,

septicemia ( Mainly of Respiratory distress )

Yound male patient who is a construction worker presenting with night sweats

and weight loss. TB

75

Urinary tract infections

cystitis/ uti symptoms, how will you confirm

a) ultrasound

b) urinalysis***

women had uti, treated with antibiotics but she returns after a week because

symptoms didnt subside, next step-

a. urine culture

b. renal u/s

c. admit and iv antibiotics

d. VCUG

GIT infections

MOA of cholera toxin o Enhance c AMP

Pregnant giardia

a. Paromomycin

Pregnant with giardia infection and diarrhea

They were asking about management

No management and observe I think (one of the options) No consistent

recommendations exist for the treatment of pregnant patients because of the

potential adverse effects of anti-Giardia agents on the fetus.

If possible, treatment should be avoided during the first trimester.

Mildly symptomatic women should have treatment delayed until after

delivery.

If treatment is necessary, paromomycin can be used as systemic absorption

is poor.

If the patient is left untreated, adequate nutrition and hydration

maintenance are paramount.

https://www.medscape.com/answers/176718-120647/how-is-giardiasis-treated-

during-pregnancy

Genital ulcer: patient with multiple, but safe sex- notice while urinating- 2 rows

of flat papules filiform, light color and texure or corona circumference- what will

u do?

a) Reassure,

b) cautery and educate on STD,

c) Antibiotics and educate on STD,

76

d) follow for malignant potential

Reassure

Man ate a meal of meat and potatoes & green beans. Next 3 hours he had watery

diarrhea. Got better with fluids and returned to work after 24 hrs. Which

organism caused this?

a) Clostridium difficile

b) Clostridium perfiringens

c) staph aures

d) yersinia enetrocolytica c

Question of an inpatient developed diarrhea what is the organism ?

question stem but it is something like this

Clostridium difficile

Patient received multiple antibiotics , now has diarrhea what is the best

diagnostics step to do ? Clostridium difficle toxin

Patient with vomiting for one hour , same presentation for her son and

husband , they ate a pie Next step? IM ranitidine , IV fluid ,IV ampicillin

Peritoneal analysis, high eosinophils, what is the cause?

a. Helminth infestation

Poisoning after fried rice what is the organism

Bacillus cerus

The type of infection that produce hepatocellular carcinoma is:

a. Epstin barr virus

b. Measles

c. Hepatitis B

indicate the acute stage of hepatitis B HbsAg + / Hbe Ag positive HbsAb + /

Hbc Ab negative, Hbs Ag + / Hbe Ab positive, Hbs Ab + /

Hbc Ab positive

STD

A man with N.Gonorrhea. Treated with Augmentin Presents

with clear urethral discharge. Gram stain showed multiple

neutrophils but no organism Cause ? Chlamydia

77

A man with N.Gonorrhea. Treated with Augmentin. Presents with clear

urethral discharge. Gram stain showed multiple neutrophils but no organism.

Cause ?

A) Resistant gonorrhea

B) Chlamydia

Male with purulent discharge, culture positive for N.G. treated with

antibio Resistant N.G

woman having one sexual partner but he might not be monogamous came

presenting with yellow - green vaginal discharge and a histology slide showing

an organism management options :

a. get her checked up for other STD

b. treat her and get her and him checked for other STD B

Direct transimission seen in sexual transmission in Chlamydia

Direct transmission is what Chlamydia by sexual contact, needle

sharing in Hep B infection, inhalation of air with TB, Cholera infected

water

homosexual had Hepatitis C 7 years ago, hepatitis A since 5 years, partner had

hepatitis A and getting treatment, what to advice him?

a. post exposure prophylaxis

b. immunoglobulins

c. Lifestyle modification

d. IgM check

e. Check IgG

Man had multiple ulcers in the gential area with rash in the palm and sole ,

picture was provide for rash in the hand what is the causative organism

Trepenima palladium

What cancer is associated with a virus? cervical

Neurologic infections

Case of meningitis what is the organism

a. Neisseria meningitidis

TB meningitis scenario, which is the following is true

a. Men more affected from female

b. No focal lesion

c. Symptoms may resolve without treatment a

Case of HIV patient with TB meningitis, Dx done by LP and CSF analysis, was

asking what is the commonest manif. associated with his condition?

78

a. internuclear ophthalmoplegia

b. something ophthalmoplegia again

c. chorioretinitis

d. Optic neuritis

e. Papilloedema

The ocular manifestations of TB in our patients included panophthalmitis,

endophthalmitis, posterior uveitis with choroidal tubercles, keratitis, and a lid

mass.

Uveitis is the most common ocular manifestation for TB

He was on an anti tb drug.so it caused optic neuritis bcz of ethambutol.

HIV man with fever, expressive aphasia and altered consciousness, CT

given and showed multiple ring enhancing lesions

toxoplasmosis, pyogenic abscess, tuberclumoa

Old man , with cns symptoms , a picture of cyst and trophozoite was provided ,

what is the cause Toxo

Ambeiasis

Tinea

Hydatid

Trypenosomiasis

Skin infections

21 University student with honey crusted lesion on lower lip tx :

A) Flucoxacillin ,

B) penicillin and flucoxacillin

C) Augmentin

D) topical fusidic acid

E) penicillin only

Burn patient, presents with a lesion, blue-green pus with fruity odour

A) Pseudomonas

B) Staph Aureus

pseudomonas

Cervical lymphadenopathy, fever, Maculopapular rash, atypical lymphocyte on

blood flim. Dx? Infectious mononucleosis

Patient domestic worker female with large cervical lymph node, night

sweat and fever, 4 kg weight loss, no sore throat, chest clear :

A) TB lympadeinitis

B) Infetious mononucleiosis

Male with venous ulcer+hyerpigmentation+superficial vein+mild non

tender calf swelling Dx and ttt? Dvt, cellulitis, ostomylitis

79

Patient with redness, swelling in the leg + ulcer of the medial side above

the ankle, hx of varicose veins , no hx of DVT what is the diagnosis

DVT or cellulitis

Patient had leg pain , erythemia , creptiations , hot , tender what is the cause ?

Staph aurse infection

Cellulitis by Streptococcus pyogene

Measles:

Lady with confined lesion to one area on her forearm herpes zoster ,

Eczema ,Scabies

Derma

Patient presented with burning pain, started from the back radiating to

the left anterior chest Herpetic neuralgia

Bone diseases

21 years old male from cameron. Visited UAE and passed the visa test medical

exam..had low grade fever since 2 months and went into fixation of his spine

fixation what is the cause

a. Pott's disease (mycobacterium bovis)

b. staph aureus infection

c. Metastatic bone lesion a

A man with osteoarthritis started exercising, had trauma to one knee, the

septic arthritis

Old man with osteoarthritis with swelling to one knee joint Septic arthritis

Diabetic patient with bilateral osteoarthritis. Advised

To walk as exercise. After one week he presented with unilateral knee effusion

and redness. Dx? Septic arthritis

Old man who has osteoarthritis and DM2 , had trauma to

his knee then developed pain , restricted movement and couldn't

extend his knee . What to do ? (From 28 Jan)

- Rest and analgesia

- Knee X-ray

- Arthrocentesis if there is fever

Others

The Haddon matrix illustrate:

80

a. host, agent and environment

b. time phase events and injuries

c. host, vector, agent and environment

Child with leukemia, started on chemotherapy, very low neutrophils, has

fever, no focal signs of infection, management do culture and wait for

results, start 3rd generation antibiotics (should be meropenem

Febrile neutropenia patient what to give o 3rd generation cephalosporin

Drug of choice

48 yrs old man sputum culture shows strep pneumonia treatment

A) azithro ,

B) Amoxicillim ,

C) Augmentin,

D) co-amoxiclav

55 years old male , presented with features of pneumonia

(Community acquired) what antibiotic to give ?

- Amoxicillin

- Azithromycin

treatment for streptococcus pneumonia

A.Amoxicillin ( if child )

B.Augmentin

C.Cefuroxime

D.azithromycin ( if adult )

An adult with typical signs and symptoms of meningeal irritation

suggestive of bacterial growth, the most appropriate treatment is:

a. IV ceftriaxone

b. Intrathecal penicillin

75 years old male presented with rash in the back representing

dermatome distribution. What should you give him?

a) Give antibiotic topical

b) Give acyclovir

c) Do blister scraping

d) Referred to dermatologist

UTI pregnant

81

a. Ampicillin

Lady with Lower UTI symptoms. Urine sample is sent for culture. What

is the TTT before results ready TMP/SMX for 7 days, nitrofurantoin

for 4 days

Pregnant pt 8 weeks, with UTI

A) Ampicillin

B) Penicillin

C) ciprofloxacin

D) Gentamicin

Man post kidney transplant , had a lung nodule then a sputum culture grew a

fungi ( battery dish was showing aspergillus fumgmantus) what best treatment

Voriconazole

Caspofungin

Linezolid

Patient has symptoms of pyelonephritis, treatment ciprofloxacin

for 14 days

infiltrate, most appropriate ABx Trimethoprim ( TMP-SMX for

pneumocystis Jiroveci )

Lady with dysuria, frequency and urgency. She was diagnosed as having

UTI. What is the best treatment as outpatient amoxicillin 14 days,

trimethoprine-sulfa for 10 days. Augmentine 10 days, erythromycin 14

days

75 years old male presented with rash in the back representing

dermatome distribution. What should you give him Give antibiotic

topical, Give acyclovir, Do blister scraping, Referred to dermatologist,

82

Immunology

Post Splene

the case Pneumococcus or H.influenza

A patient who is a known case of chronic renal failure is a candidate for

which vaccination:

a. hepatitis A

b. Meningitis

c. Pneumoccocal

d. Himophilus influenza B

Child with recurrent infections. Labs showing deficiency of IgA, IgM, IgE etc

and decreased circulating mature B lymphocytes:

a. CVID

b. CSID

c. Brutons immunoglobulinemia***

ans: c

Contact dermatitis is which type of hypersensitivity ?

Type 4

Type 1

Type 2

Type 3

Eczematous rashes , ecchymosis, Suspicious of child abuse as brother also died

from fungal encephalitis /meningitis

a) Wiskott Aldrich**

b) SCID

Patient developed allergic reaction after eating shrimp, management

Epinephrine

83

Microbiology

Micro Man ate a meal of meat and potatoes & green beans. Next 3 hours he had

watery diarrhea. Got better with fluids and returned to work after 24 hrs. Which

organism caused this?

a) C. diff

b) c. perf

c) staph aures,***

d) yersinia enetrocolytica

organism.

a) salmonella typhi***

b) shigella

c) Entamoeba histolytica

d) yersinia enetrocolytica.

e) clostridium difficile

Micro most likely organism to cause infection post splenectomy.

A.Klebsiella

B.E.coli

C.Streptococcus pneumonia

Micro Old patient was present with community acquired pneumonia and was

started on Levofloxacin. Now patient is improving. This medication act on:

a. Gram positive and negative but not atypical

b. Gram positive and negative and atypical***

c. Gram negative and atypical but not positive

Micro Patient had injury with glass 3 days ago, no presented with Wound

infection, catalase postive, non motile,shows white spots and hemolysis on blood

agar, pleomorphic,,,

a) actinobacter pyogene

b) bacillus cerus

C) listeria monocytogenes

The patient with wound infection and pus after cutting himself with a glass, the

culture of the wound results on many types of agars, Gram +, pleomorphic

arranged in chain, Catalase +

What is the organism ?

In exam it was mentioned that it is non motile so listeria is non motile

84

a. Actino (corynobacterium) something

b. Actinomyces (corynobacterium) Pyogenes**

c. Bacilus cerus

d. Listeria monocytogenes

Micro Patient presented with H.pylori?

a. Gram negative rod

b. Gram positive rod

c. Spiral gram negative ***

d. Gram positive cocci

e. Gram negative cocci

Micro Patient presented with vomiting. On examination palpable purpura. LP

show no organism.. blood show kidney shape organism. MacConkey negative

a. Neisseria gonorrhoeae

b. Neisseria meningitides**

c. Neisseria lactamica

Which cancer can be caused by virus. Lung, liver, cervix

HPV 16, 18 Cervical cancer

HBV hepatocellular carcinoma

Cholera pathogenesis:

cAMP stimulation

85

Nephrology

Renal failure

CKD and rash, bipedal edema and dry

a) emoliants

b) gabapentin***

Acute renal failure in a tourist in the dessert?

Hypoperfusion

Acute renal injury in Diabetic person after IV contrast CT. HCO3 was given

before giving Contrast. Good Control of DM with 6.8% HbA1C , baseline GFR

>60% when checked 2 months ago. Risk factor for the accelerated renal injury?

2. Diabetic nephropathy**

3. Baseline GFR

4. Givin HCO3

Another qsn about creatinine being excerted by the kidney forgot the exact qsn

Man on dialysis noticed absent thrill at home 2 days prior, came, thrombectomy

done but failed, another av fistula is formed. what would have been done to

prevent failure?

a) Streptokinase

b) aspirin

c) use other way for dialysis

d) catheter

e) monitor thrill

Old man who is on dialysis , he underwent angioplasty for his AV grafting which

was unsuccessful but it failed . now ANOTHER AV GRAFT IS BEING

PREPARED . How could you prevent the failure of the graft ?

a. Monitor flow, thrill

b. Streptokinase

c. Aspirin

Patient known CKD with lab showing high phosphate how you will treat

Phosphate binder

86

Glomeronephritis

Patient one week, increased periorbital edema, facial swelling and

puffiness, scrotal edema, what best investigation that might show

abnormality Urinalysis ( will show RBC cast), Echo

Child presented with blood in urine and deranged renal function test and edema ,

ASO titer was high PSGN

Patient with rusty urine, untreated URTI. Dx? PSGN

Child brought by his mother. has periorbital edema, improves

during the day .then had hematuria .red cell cast. .mother reported that

child has URTI 10 days ago that not treated, what is the most likely

diagnosis?

*IgA nephropathy *glomerulonephritis

Patient presented with periorbital edema, legs edema, proteinuria,

increase triglyceride, what is the diagnosis Nephrotic syndrome

Boy with bilateral edema pain with pressure on edema diagnosis?

nephrotic syndrome

Child with nephrotic syndrome and renal vein thrombosis what is the expected

lost protein ? Anti thrombin 3

Lady came with periorbital edema and low serum albumin

(direct picture of nephrotic) , what is the diagnosis ?

- Nephrotic

Treatment of minimal change disease

prednisolone

Patient with diagnosed with nephrotic syndrome after evaluation of

proteinuria, which of the following showed to decrease the proteinuria ?

ACE-I

35 years old man had a history of rhinitis and hay fever ( i think ) came

presenting with renal symptoms ? These renal symptoms occur every time he

has a URTI.

1. IgA nephropathy/***

2. Post streptococcal GM

3. Thin basement membrane

4. Good pasture syndrome

ANCA +ve, renal and respiratory symptoms, cross section was taken. From the

kidney (showed blood)?

a.

b. Granulomatosis with polyangiitis

Linear deposit in IMF from kidney biopsy o Good pasture

Man with chronic sinusitis, high creatinine and 3+ blood on dipstick

87

Wegener

Patient presented with No previous URTI or abdominal pain

With low complement 3 and 4... Hepatitis C positive. Hepatitis B negative..?

palpable purpura

a. IgA nephropathy

b. Membranous nephropathy

c. Membranoproliferative GN

d. Mixed cryoglobulinemia d. Mixed cryoglobulinemia

https://step2.medbullets.com/rheumatology/120645/mixed-cryoglobulinemia

Patient with cough and SOB. Increase in urea, creatine and renal biopsy

showing focal necrotizing vasculitis, Bilateral interstitial nodules in CXR,

ANCA +ve what could be the other respiratory symptoms?

a. Hemoptysis***

b. Angina

c. Hemothorax

d. Hemorrhagic pericarditis

Others

polycystic kidney disease which statement explains it the best?

A.Autosomal recessive

B.95% the son will have the disease

C.x linked disease

40s years old man has polycystic kidney disease, what is correct about

this disease Autosomal recessive (Should be Autosomal dominant),

X-linked (there was no autosomal dominant from the choices)

Adult male with PKD. which of the following is true AR inheritance

(dominant!), his son will have 95% chance of having same disease

62 years old man with polycystic kidney disease , on further history

taking it reveals that he had it since childhood . What is true about

this condition ?

- His 36 years old son will have disease in 90-95%

- It is autosomal recessive

- Associated with lung cysts

- X- linked disease

Autosomal dominant the age of onset will be above 30, while

Autosomal recessive will be since childhood

A man lost his way in the dessert for 2 hours . He was found by a

passerby and brought to emergency . His creatinine is 3.2 what is

the most likely of this creatinine ?

- Rhabdomyolysis

88

Patient has HTN, DM, takes many drugs, one of them is NSAID,

underwent surgery, bleeding happen, BP dropped , 8 units of blood given,

renal functions test showed renal failure and urine microscopy showed

granular casts NSAID induced renal failure, Tubular necrosis

female with High bp (200+/100+) and positive fundoscopy findings.

What is the most likely cause and answer is renal artery stenosis. (other

choices were coarctation but both limbs had same BP, Aortic abd Aneurysm

which is clearly wrong and essential HTN which is unlikely based on her

case)

Old male with high BP ( 200 / 150 ), on abdominal examination a bruit

was heard during auscultation. Dx ? Renal Artery Stenosis, Aortic aneurysm

Patient presented with renal stone after analysis it shows that normal uric acid all

other labs are normal but low citrate how to treat

a. Increase hydration

b. Allopurinol

c. Citrate****. (potassium citrate

d. Penicillamine

89

Neurology

Cerebrovascular diseases

45 yo Patient who develops in only 1 day hemiplegia in his lower limbs power

2/5, arreflexia,

o Anterior spinal cord syndrome

o Spinal cord compression

o Infection

o Tumor

patient presents with TIA. how to prevent further future episodes?

A.Aspirin

B.clopidogrel

C.warfarin

Patient presented with right sided facial weakness and inability to express

himself , with loss of sensation over the big toe , picture of brain areas was

provided , choose the possible affected area ?

A man with recurrent bleeding from the nose and also buccal Telangiectasia and

gum bleeding, when having dental procedures, MRI brain was shown ECR was

high, showing some hypodense area, whats next step?

a. Hemicraniectomy

b. Iv methylprednisolone

c. Angiogram

African lady with many symptoms (symptoms of infection) in which she took

aspirin for. Parents brought her to ED due to sudden disturbed behavior. Her

Brother is diagnosed with schizophrenia. ANCA positive.

What is the underlying cause of her disturbed behavior ?

a. Infarct

b. Cerebral vasculitis

c. Infection

lady with upper and lower limbs weakness. .incoherent speech but got better

after 2 hours aspirin

Old lady with previous infarction..she was confused BP normal developed

urinary incontinence just during admission what is the cause of her incontinence

, US done which showed residual volume 80 ml What is the diagnosis?

a. Normotensive hydrocephalus

b. Functional incontinence

c. Overactive incontinence

90

Patient with weakness in lower and upper limbs and in speech and other .intact

sensation

o Cerebral tumor

o Motor neuron disease

o Anterior column system

o Spinal stenosis

Patient brain dead what reflex will be preserved

a. Gag

b. Corneal

c. Pupillary

d. Spinald. Spinal

https://www.ncbi.nlm.nih.gov/pmc/articles/PMC2729975/

Truck driver with sensory loss in the upper and lower limbs. Has DM and other

problems? What is the cause of sensory loss? DM

Dementia

Old man brought by his son complaining his father is having memory

loss for one-year. He was independent but recently he forgot where his car

was? He also told his son that neighbors trying to poison him. Diagnosis

A) Dementia

B) Schizophrenia

C) Depression

70 year old man brought by his son to clinic due to memory loss.one

episode of losing way and son has noticed that he was calling the

grandchild by son's name. Also mentions he is happy and believes it was

A.Alzheimers

B. age related dementia

Old man, decreased interaction with his grandchildren, his family has to

bring him and

b. age related dementia c. major depression

old man with features of dementia and one episode of thinking his

neighbours are poisoning him dementia, Schizophrenia, Delirium

63 years female patient in nursery, you asked to examine her, you

heard her screaming from outside to take her out from this prison, she

was confused, refused to take medications and asked to be alone

Delirium

70 year old man brought by his son to the clinic due to memory loss. One

episode was of losing his way. His son noticed that he was calling the grandchild

by the son's name. He also mentions that he is happy and believes it was only

91

agnosis? Age

related dementia

72 years old patient brought by his son for evaluation of dementia, he

had decreased concentration, apathy , depressed mood. Patient was

euthiymic on examination. He had slow thinking, low verbal fluency,

inability to recall recent events and motor signs. What is the type of

dementia?

a) Alcohol dementia

b) Alzheimer dementia

c) Huntington dementia

d) Lewy body dementia

e) Pseudo- dementia

Elderly lady admitted with Myoclonus weakness, incontinence, ?speech defect,

valvular HD history, confused, not eating. Patient had low albumin. What is the

cause of her functional decline?

a) end stage dementia

b) end stage renal

c) end stage liver

d) age related deliability

e) end stage stroke a

as confirmed with an IM physician. The case is a presentation of Corticobasal

dementia

Reversible cause of dementia ?

- Pellagra

- Creudzfelt Jacob

- Multi-infarct dementia

Neurological syndromes

Question about Zellwegers Syndrome. What are the macromolecules that

accumulate in the brain?

a. very long chain fatty acids****

b. ketones

c. lactate

d. glucose

10 year old girl with evening weakness , ptosis , she is better in the morning

Myesthenia gravis

lady complaining of inability to comb her hair, feels pain when driving, bilateral

proximal muscle weakness. Diagnosis?

Myasthenia Gravis

Patient with inability to raise his arms up, ptosis, worsen as the day goes

92

by. Dx Myasthenia gravis

myesthenia

graves, Spiral polymyosistis

pathophysiology of myasthenia graves NMJ disease

female patient how develops weakness in the jaw and blurred vision specially at

the end of the day,

Asking about the best initial test

o Neck ultrasound

o Brain MRI

o Chest CT

o Echo

o CSF

Case of ptosis and diplopia , looks like myasthenia clinical , what is the next test

Endorphin test

Female in ICU cannot lift shoulders and could not move the legs, what is

the correct test for Dx Edophonium ( for MG ) 24h VMA, Creatinine

and BUN,

Case of Motor neuron disease, with lower weakness, speech affected, sensation

a. Neuromuscular junction disease

A 47 years old male patient was hospitalized for the removal of

gallbladder. On the second day after surgery the patient was ataxic and has

memory problem. On examination the patient had diplopia. What could

prevented this? a. Thiamine b. Vitamin B6 c. Vitamin B12 d. Niacin

( Wernicke Korsakoff syndrome

A man had loss of motor functions, and areflexia of the knee and ankle but the

planter reflex was preserved , His history is insignificant except for a one week

(Plantar reflex is upper motor is intact) Guillain barré Syndrome

Patient with ascending weakness, arefflexia of patella and ankle I think

with LP showing 8 lymphocytes and 158mg proteins something like that

GBS Gullane barrie syndrome

Case of a man who has decreased pupil size, a drooping eyelid, and decreased

sweating on one side of his face. What is the diagnosis?

a.

b.

c. Superior Vena cava syndrome

d. Lambert Eaton syndrome

93

Patient with loss of sensation on left side of face with vesicle eruption on face

and ear, with pain in tympanic membrane and ear canal. All investigations and

imaging studies were normal.

a.

b. Acoustic neuroma

c. Herpes zoster oticus

Female patient with LL weakness, then resolved, after while developed diplopia

MS

Headache disorders

time. No other symptoms. No aura or photophobia. Dx? Tension type headache

Female patient with bitemporal headache for 2 years. She used to take

paracetamol 600mg and ibuprofen 400mg a day which used to help, they stopped

working after 1 year and the headache is worsening. No red flags. What kind of

headache does she have? Tension headache

Hx of headache, unilateral pulsating. No symptoms of aura mentioned

. Patient likes to be left alone and calm when she has the episode. Dx?

- Migraine

- Tension headache

- Cluster headache

26 years old with history of recurrent headache, C/O severe headache on

the right side of his head, started as dull aching, then became more

severe, throbbing in nature and causes nausea, vomiting and photophobia,

he developed hemiparesis on the left side of his body temporal

arteritis, migraine, cluster, subarachnoid hemorrhage

Lady, came complaining of a headache and morning vomiting. Her history was

insignificant except for a time 4 months ago, where she got hit in her head while

a. Subdural hematoma

b. brain tumor

c. meningitis

or

cardiovascular support . Her husband said that recently she is

having regurgitation and vomiting specially at morning . What is

the most likely diagnosis ?- Intracranial tumor

Guy with headache vision changes, vomiting and problem with

balancing? Red *brain tumor

Two question of intracranial hypertension with red flags, morning

headache , morning vomiting , one case had seizure episode , and the

94

other regurgitation vomiting What's dx? Intracranial tumor ((Diagnosis

of intracranial tumor , red flags of intracranial tumor))

Which tumor metastasizes more to the brain? skin

Case of aman with Jaw numbness and tingling, change of voice after talking for

long over the phone. Which investigation should be done for him ?

a. Ultrasound for the neck

b. CT scan

Trigeminal neurolgia Carbamazepine

Depersonalization associated with? Temporal lobe epilepsy

Elderly Patient alcoholic with generalized vague pain, nothing was found on

examination. No history of any medical condition. What is the cause?

a. Alcoholic dependence

b. Factitious

c. Malingering

d. Pain related to Substance abuse

e. Pain related to medical condition

Movements disorder

A lady with right sided tremor , slowly moving what is the cause ? Parkinson

disease

best following is feature of Parkinsonism?

* Spasticity

* Jerky movements

* resting tremor improves with work

Case of parkinsonism: shuffling gait, resting tremor. Rx? Levidopa + carbidopa

Diabetic patient Who was taking parkinson medication (methyldopa) And

metformin, he develops hyperprolactinemia, what caused it?

a) Methyldopa**

b) Metformin

c) Dopamine agonist effect

Man in his 50s compiling a resting tremor that he can control for a few mins then

it continues , you also noticed the same during the visit. What is true?

a. Carbidopa is effective treatment

b. Men = women

c. Patient will not be able to voluntarily control it

with action . Include the head . What is the cause ? o Parkinson

o Normal for his age

o Essential

Parkinson

95

You might also like